Inv Plng

  • November 2019
  • PDF

This document was uploaded by user and they confirmed that they have the permission to share it. If you are author or own the copyright of this book, please report to us by using this DMCA report form. Report DMCA


Overview

Download & View Inv Plng as PDF for free.

More details

  • Words: 28,180
  • Pages: 78
Investment planning 1 Mark Questions 1. The bid-asked spread is best described by which one of the following statements? (a) (b) (c) (d) (e)

It is the broker's commission. It is the dealer's gross income from a transaction. It is larger for illiquid securities than it is for liquid securities. All of the above are true. None of the above.

2. How do T -bills pay interest to their investors? (a) (b) (c) (d)

Coupon interest. Possible price appreciation above their discounted price. T-bills pay no interest. Difference between issue price and face value

TBs are issued at a discount to the public and the RBI for enabling the Govt to meet the needs for shortterm finance.They are issued by following an auction system. 3. Which of the following are characteristics of money market securities? (a) They are issued by the Government, municipalities and large corporations that have highquality ratings. (b) All have terms to maturity that are 270 days or less. (c) All tend to have large amounts of purchasing power risk. (d) Both a and b. (e) Both band c. T Bills have a tenure of 364 days and hence option (b), (d) and (e) are incorrect. Money market instruments are Tresury Bills, certificates of Depost, Commercial Paper and Repos. The money market is dominated by Government, Financial Insititutions, Banks and corporate.Individual Investor scarcely participate in the money market directly. Hence option (c) also in incorrect. 4. A basis point is which one of the following? (a) (b) (c) (d)

One Rupee, Re.1 One percentage point, 1 percent One paisa, Re.0.01 One one-hundredth of one percentage point, 0.01 of 1 percent,

5. Government bond is best described by which one of the following statements? (a) (b) (c) (d)

It has no voting privileges. It receives no cash dividends. It may be resold at any time. All of the above.

6. Equity shareholders have which of the following rights? (a) They can legally demand information from a corporation in which they are a shareholder and thus gain access to its books. (b) They can vote for the common shareholders' dividend. (c) They can vote for the preference shareholders' dividend. (d) All of the above. 7. Preference shareholders receive priority over common stockholders with respect to which of the following? Investment Planning – 19Oct 07

1

(a) Dividends cannot be paid to common stockholders unless the preference stockholders receive their stated dividend. (b) In the event of bankruptcy and liquidation, the preference shareholders are paid before the common shareholders. (c) Preference shareholders get to elect the Chairman of the corporation's Board of Directors. (d) Both a and b are true, but c is false. (e) All of the above are true.

8. Which one of the following equations correctly defines the dividend yield (y) from a share of common stock? a) b) c) d)

y = (purchase price) + (cash dividend, if any) / purchase price y = (price change) + (cash dividend, if any) / purchase price y = price change /purchase price y = cash dividend (if any) / purchase price

9. A preference share is a) b) c) d) e)

Pays fixed dividend A marketable security A debt security Both a and b All of the above

10. Which of the following statements best describes the convertibility of preference share? (a) Some issues of preference share may be converted into common share at the option of the investor any time and at a conversion ratio that never changes. (b) Some issues of preference share may be converted into common share at the option of the investor within a limited number of years after the preferred stock is issued. (c) Some issues of preference share may be converted into common share at the option of the investor only after a specified number of years have elapsed since the preference share was initially issued, (d) All of the above are true. (e Preference share is never a convertible security. 11. Which one of the following statements best describes corporate bonds? (a) Bond investors are creditors of the corporation. (b) The majority of bonds make coupon interest payments once per annum. (c) Both a and b are true. (d) None of the above are true. 12. A debenture trust deed is best described by which one of the following statements? (a) It is a legal contract describing the rights of specific bondholders. (b) It describes the duties of the Trustee. (c) Both a and b are true. (d) None of the above are true. 13. Debenture trust deed may contain protective clauses dealing with which of the following topics? (a) (b) (c) (d)

Collateral A .sinking fund All of the above None of the above

14. The quality ratings of a corporation's bond issue are primarily determined by which of the following? V (a) The level and trend of the issuer's financial ratios (b) The level and structure of interest rates (c) The issuer's financial condition and the indenture contract that governs the issuing firm Investment Planning – 19Oct 07

2

(d) All the above 15. One bond with an AA-grade rating might pay a higher yield-to-maturity than another AA-grade bond issued at a different time by the same corporation because of which of the following reasons? (a) Bonds with longer maturities always pay higher rates of interest than similar bonds that have shorter maturities. (b) The bond market is sometimes irrational and evaluates the riskiness of some bond issues erroneously. (c) One bond issue is a secured one whereas the other issue is unsecured. (d) All of the above

16. Which of the following bond quality ratings applies to default-free bonds? (a) (b) (c) (d)

AAA. AA. Both b and a are default-free. None of the above is default-free.

17. A security will not earn the yield-to-maturity that was promised when the security was purchased if which of the following conditions occurs? (a) The issuer defaults on either the interest or principal payments. (b) The investor sells the security prior to its maturity date. (c) Cash flows from the security paid to the investor prior to its maturity date are held in cash or spent on consumption goods rather than reinvested. (d) All of the above are true. (e) None of the above are true. 18. An investor that employed a naive buy-and-hold strategy would be employing a passive investment management strategy. (a) True (b) False 19. An index fund is best described by which of the following? (a) A mutual fund constructed to achieve a particular investment goal (b) A portfolio that attempts to match the performance of some stock market index by investing in the same stocks and in the same proportions as those that comprise the selected market index (c) Both of the above (d) An investment portfolio that appreciates in value at least as rapidly as some inflation index (such as the Consumer Price Index, for instance) (e) Both a and d 20. The weights used in constructing a value-weighted stock market index are best described by which of the following statements? (a) Equal weights are assigned to every security in the index. (b) The weight assigned to each stock is proportionate to its price per share. (c) The share price of every stock in the index is multiplied by the number of shares outstanding to determine the weight of that issue based on its total value stated as a proportion of the aggregate market value of all the stocks in the index. (d) The weight assigned to each stock in the index is proportional to the number of shares that issue has outstanding stated as a proportion of the aggregate number of shares outstanding for all issues that comprise the index. (e) Both a and d are true. 21. All stock market indexes are most accurately characterised by which of the following statements about the degree to which they covary together? (a) They are perfectly positively correlated. (b) They are highly positively correlated. Investment Planning – 19Oct 07

3

(c) They are uncorrelated. (d) They are negatively correlated. (e) It is impossible to generalize, some are highly positively correlated and some are negatively correlated. 22. Depreciation must be entirely omitted from a firm's net profit in order to determine how much cash flow the firm generated. (a) True (b) False

23. When a firm pays creditors the transaction does not affect the equity capital shown in its balance sheet in any way. a) True b) False 24. A primary issue of bonds or stock would increase both sides of the issuing company‟s balance sheet by the same amount. a) True b) False 25. The retention rate equals 100 percent less the percent of the corporation's earnings paid out for cash dividends. a) True b) False 26. If a corporation has preference share outstanding its book value per share equals its total net worth divided by the number of shares of common stock plus preference shares it has outstanding. a) True b) False 27. Which of the following ratios will increase as a firm uses more financial leverage? (a) (b) (c) (d) (e)

The times-interest-earned ratio The debt-to-equity ratio The inventory turnover Both a and b Both a and c

28. Which of the following factors tends to increase the growth rate of a corporation? (a) (b) (c) (d) (e)

External borrowing Increasing the retention rate Increasing the rate of return on equity Both a and b All of the above

29. A company has total assets of 2,000,000. It has 700,000 in long-term debt. If stockholders' equity is 900,000, what is its total debt to total asset ratio? (a) (b) (c) (d) (e)

45 percent 47 percent 59 percent 52 percent 55 percent

30. A corporation had a total debt to total asset ratio of .4, total debt of Rs.200,000, and net income of Rs.30,000. Determine the corporation's return on equity. Investment Planning – 19Oct 07

4

(a) (b) (c) (d) (e)

8 percent 9 percent 10 percent 12 percent 14 percent

31. Assume the following information: stockholders' equity = Rs.2,000; shares outstanding = 40; market price to book value = 2. Determine the market price for the firm's common stock. (a) (b) (c) (d) (e)

Rs.75 Rs.100 Rs.110 Rs.115 Rs.117

32. Which of the following is a source of funds? (a) (b) (c) (d) (e)

An increase in inventory An increase in accounts receivable An increase in investments An increase in accounts payable None of the above

33. Assume the correlation coefficient r between the rates of return from these two cement sector shares, say, A and G was +0.9. If you took a long position in G and a short position in A (or vice versa) of exactly equal value you would be perfectly hedged. (a) True (b) False 34. Imperfect hedges occur when either the quantities sold short and bought long are out of balance, or the purchase and short sale prices differ. (a) True (b) False 35. What characteristics do the long and short positions have in common? (a) The potential profits from a long position and the potential losses from a short position are both infinite if the price of the underlying security rises to infinity. (b) There is a one-to-one correspondence between movements in the price of the underlying security and the investor's profits (c) Both b and a are true. (d) The investor loses money if the price of an asset held in either position declines. (e) Both a and d are true 36. Which of the following statements correctly describes a speculator‟s (short seller) profits? (a) The per share profit is limited to an amount equal to the price at which the shares were sold short. (b) The short seller earns Re.1 profit for every Re.1 fall in price of the security. (c) Short selling can help arbitragers earn profits. (d) All the above are true. 37. Selling securities short is useful in which of the following activities? (a) Speculating (b) Hedging Investment Planning – 19Oct 07

5

(c) Arbitrage (d) All the above 38. Investors‟ motives are not correctly described by which of the following statements? (a) (b) (c) (d) (e)

Having long positions indicates that the buyer is bullish. Uncovered short sellers expect the market to be bearish. Hedgers are always bullish. Both a and b are false All of the above are false

39. Why do share prices usually drop when news about decline in a company‟s earnings per share is reported? (a) Because a reduction in a earnings means that the firm has less money with which to pay dividends and therefore the market fears a reduction in the company's future dividends. (b) Because the share market anticipates that a decreased level of earning power might be the indicator of default and perhaps even bankruptcy. (c) The statement is false. Share prices do not usually react to announcements about current earnings. (d) Both a and b are true. 40. Interest-rate risk is defined by which of the following statements? (a) Fluctuations in the coupon interest rates from one bond issue to the next (b) Fluctuations in the market prices of bonds as their prices move inversely to the prevailing market interest rates (c) The variability of returns as a result of fluctuations in market interest rates (d) Both a and b (e) All of the above 41. Assume that you are an investment adviser and one of your clients, on your advice, invested Rs.100,000 in Treasury bonds due to mature in 2 years. If your client becomes worried that a general increase in the level of interest rates will reduce the market value of his bond portfolio, what should you say to allay your client's fears? (a) You could assuage your client's fears by claiming you foresee only stable interest rates ahead. (b) You could instruct your client to liquidate their portfolio of Treasury bonds and reinvest the proceeds in a bank. (c) Both a and b are true. (d) You could tell your client not to worry because the market prices of short-term bonds do not fluctuate very much. 42. Assume you are an financial advisor and one of your clients reads something about interest-rate risk and is worried that if market interest rates declined her coupon interest income will likewise decline. His bond investments have maturities ranging from 15 to 30 years. What advice is appropriate for this client? (a) Tell the investor to liquidate her coupon-paying bonds and reinvest the money in zero coupon bonds. (b) Tell your client not to worry, her coupon income will not vary until her coupon bonds mature in 15 to 30 years. (c) Both a and b are true. (d) The client need not worry if market interest rates are expected to rise because coupon rates vary inversely with market interest rates and therefore her coupon interest could increase. (e) All the above are true. 43. Calculate the (1) expected rate of return, E(r), from the probability distribution of returns below for the ABC common share. FIVE POSSIBILITlES

RATES OF RETURN

i=5 -0.5 = 50% i=4 -0.1 = -10% Investment Planning – 19Oct 07

PROBABILITY 0.1 0.25 6

i=3 i=2 i=1

0.2 = 20% 0.5 = 50% 0.9 = 90%

0.3 0.25 0.1

Total 1.0 The expected rate of return for ABC is which one of the following? (a) The E(r) is 5 percent. (b) The E(r) is 20 percent. (c) The E(r) is 5 percent. (d) The E(r) is 10 percent. (e) The E(r) is 12 percent. 44. Which one of the following statements describes the phrase „risk‟? (a) The phrase total risk is synonymous with the variability of return from an asset. (b) Bond quality ratings essentially measure the probability that an issue of bonds falls into default. (c) Although Treasury bonds are free from default risk they nevertheless contain substantial amounts of interest-rate risk. (d) Both a and b are true. (e) All the above are true. 45. If ABS‟s price is Rs.40 per share and its current dividend of Rs.3.85 per share, which is growing at a 7 percent rate per year, determine its required return? (a) (b) (c) (d) (e)

16.2 percent 15.1 percent 16.6 percent 17.3 percent 18.2 percent

46. If ABS‟s pays dividend of Rs.3.85 per share which is growing at a 7 percent rate per year and is expected to grow at the same rate in future. Its required rate of return is 14.5% Determine its share value. (a) (b) (c) (d) (e)

52.48 49.25 54.93 55.75 47.26

47. A company has current earnings per share of Rs.6. Assume a dividend-payout ratio of 55 percent. Earnings grow at a rate of 8.5 percent per year. If the required rate of return is 15 percent, what is its current value? (a) (b) (c) (d) (e)

Rs.51.33 Rs.55.08 Rs.57.02 Rs.52.05 Rs.50.75

48. The value of an option tends to increase as the volatility (or risks) of the underlying asset increases. a) True b) False 49. If you purchase a put option, you are expecting the value of the underlying asset to increase. a) True b) False Directions for Questions 50 to 58: Use the information given in the table below to answer the questions. Ignore taxes and transaction costs. Time to Maturity Investment Planning – 19Oct 07

7

Share A B C

Current price 52 40 35

Exercise price 50 45 30

Call Premium 3 months 6 months 3 4 1 1.25 6 6.3

Put Premium 3 months 6 months 0.35 1.05 5.5 6.00 0.45 0.65

Each contract is equal to 100 shares. .

50. If you purchase one 3-month call contract on A, what profit or loss will you make at the maturity date if the price of A at that time is Rs.57? (a) (b) (c) (d) (e)

Rs.200 Rs.400 -Rs.460 Rs.500 Rs.560

Exercise Price X = 50. Premium Paid= Rs.3 Total Cost = Rs.53 *100 = Rs. 5300. This is the amount for which you have the option of buying the shares today. Market Price at the time of expiry of option = Rs.57 * 100 = Rs. 5700 Thus 5700 – 5300 = Rs. 400 is the profit for you.

51. If B's price is Rs.35 at the maturity of the 6-month option, determine the value of five 6-month put contracts at their maturity date. (a) (b) (c) (d) (e)

Rs.2,000 Rs.5,700 Rs.8,200 -Rs.4,000 Rs.3,600

52. If you had purchased five 3-month call options of C and the price of C's share is Rs.32 at maturity. Determine your profit or loss on the investment. (a) (b) (c) (d) (e)

Rs.1,000 Rs.1,500 -Rs.2,000 -Rs.4,000 -Rs.500

A 3 month call option for C will come at (X)Rs.30+ (Premium)Rs.6 = Rs.36. for five lots of 100 the price at which you have the right to buy the shares is Rs. 36*100*5 = Rs.18000 Now the price of the share at maturity is Rs.32*100*5 = Rs. Rs.16000 Thus Rs.18000- Rs.16000 = Rs. 2000 is the loss on the investment. 53. If you had purchased five 3-month puts on C, what would your profit "or loss position have been at maturity if the share's price was Rs.32? (a) -Rs.225 (b) -Rs.400 (c) -Rs.600 (d) Rs.400 (e) Rs.600 Here the exercise price of the put option is (30* 100) – (.45*100) = Rs. 2955 Now is the current market price is Rs. 32*100 = Rs.3200 then you will incur a loss of Rs. 3200-2955 = Rs. 245 on this transaction. Investment Planning – 19Oct 07

8

54. Your client wrote five 6-month call options on B's share. What is his profit or loss on the options at maturity if the price of B at that time is Rs.43? (a) (b) (c) (d) (a)

Rs.625 -Rs.600 Rs.400 Rs.300 '(e) Rs.200

55. If your client had written five 6-month put options on B, what would his profit or loss have been at the maturity of the options if the share price was Rs.43 per share? (a) (b) (c) (d) (e)

Rs.1,000 Rs.2,000 Rs.1,800 Rs.1,500 -Rs.500

56. Which of the following options are in the money? (a) (b) (c) (d) (e)

A's 3-month call B's 6-month put C's 6-month put a and b None of the above

57. If an investor is bearish on a share, buying a put is usually better than selling short because (a) The holder's losses can be no more that the put premium if the share price rises, but the short seller's losses could be unlimited in this situation. (b) The short sale will become worthless after a short period of time but the put will not become worthless. (c) The short seller must pay any dividends paid by the security the short seller borrowed. (d) a and b. (e) a and c. 58. Call option-premiums for a given asset tend to increase when (a) (b) (c) (d) (e)

The price of the underlying asset decreases. The volatility of the underlying asset decreases. The time to maturity of the option increases. a andb. None of the above.

Directions for Questions 59 to 63: Choose the correct option 59. The Black Scholes model cannot be used to determine the overall market value of a firm. (a) True (b) False 60. When a firm's dividend payment is included in the Black Scholes model, the value of a put decreases. (a) True (b) False 61. Dividend payment on a firm's common share tends to lower the value of a call option on the firm's equity. Investment Planning – 19Oct 07

9

(a) True (b) False 62. Option prices that are calculated with the Black Scholes model are not very sensitive to changes in the asset's standard deviation of returns. a) True b) False 63. The option values calculated with the binomial model will approach those calculated with the Black Scholes model for a given period of time as we divide the fixed time period into smaller and smaller units. a) True b) False Directions for Questions 64 to 73: Using the information given below, answer the questions with the Black Scholes option pricing model. The options on the share of National Corporation have the following values: S = Rs.55 k = Rs.52 R = .10 u = .33 and T = .4 Assume that no dividends are currently being paid. 64. What is the value of a call on the National Corporation? (a) (b) (c) (d) (e)

Rs.6.50 Rs.6.80 Rs.7.17 Rs.8.05 Rs.8.35

65. If the call is undervalued, what approach should an investor follow? (a) (b) (c) (d) (e)

Buy 1,000 calls, sell short 712 shares of National share. Buy 1,000 calls, sell short 600 shares of National share. Buy 500 shares of National, sell 1,000 calls. Buy 714 shares of National, sell 1,000 calls. Buy 600 shares of National, sell 1,000 calls

66. Determine the value of a put on National's share. (a) (b) (c) (d) (e)

Rs.1.75 Rs.2.13 Rs.2.65 Rs.2.95 Rs.3.15

67. If the put on National's share is overpriced, what should an investor do? (a) (b) (c) (d) (e)

Sell 1,000 puts for every 400 shares sold. Buy 1,000 puts for every 300 shares purchased. Sell 1,000 puts for every 288 shares sold. Sell 1,000 puts for every 350 shares purchased. Buy 1,000 puts for every 400 shares sold.

68.Assume the National Corporation is paying dividends at the rate of 4 percent per year. Determine the price of a call. (a) Rs.5.57 (b) Rs.5.85 (c) Rs.6.03 Investment Planning – 19Oct 07

10

(d) Rs.6.35 (e) Rs.6.74 . 69. Determine the price of a put on National's share with a 4 percent annual dividend. (a) (b) (c) (d) (e)

Rs.2.56 Rs.2.85 Rs.1.95 Rs.2.40 Rs.2.96

70. If the price of a National call is Rs.6.41, determine the implied standard deviation of returns without dividends. (a) (b) (c) (d) (e)

.37 .40 .25 .35 .18

71. If R rises to .12, determine the price of a call on National's share if the other inputs do not change. Assume no dividends. (a) (b) (c) (d) (e)

Rs.6.80 Rs.7.25 Rs.7.61 Rs.7.90 Rs.8.10

72. If T = .5, determine the value of a put on National's share if the other inputs do not change. Assume no dividends. (a) (b) (c) (d) (e)

Rs.1.50 Rs.1.75 Rs.1.91 Rs.2.51 Rs.2.30

73. If k = Rs.54, determine the value of a call on National's share if the other inputs do not change. Assume no dividends. (a) (b) (c) (d) (e)

Rs.5.86 Rs.6.19 Rs.6.30 Rs.6.42 Rs.6.76

74. With two assets, as the correlation coefficient between the two assets is reduced, the portfolio's risk is reduced. a) True b) False 75. A 50-asset portfolio has ----- unique covariance terms. (a) (b) (c) (d)

2499 1449 1225 none of the above

76 In the two-asset case, the portfolio risk-return possibilities are nonlinear when the correlation between 11 Investment Planning – 19Oct 07

the asset returns is less than + 1. a) True b) False Directions for questions 77 and 78 The ex post returns of 2 shares are: YEAR RETURN A 1991 20% 1992 -10 1993 15 1994 17 1995 19

Return B 30% -20 18 10 5

77. Calculate the covariance of returns. (a) (b) (c) (d) (e)

220.32 -420.11 145.22 270.36 162.08

78. Calculate the correlation coefficient from the above information. (a) (b) (c) (d) (e)

.540 .869 .923 .758 .697

Directions for questions 79 and 80: Refer to the data below Two companies, M and N have the following risk and return statistics: Standard deviation (M) = 18%; Standard deviation (N) = 30% Expected Return (M) = 14%; Expected Return (N) = 19% Correlation coefficient = 0.28 79. Determine the risk of a portfolio of 25 percent M and 75 percent N. (a) (b) (c) (d) (e)

18.25 percent 30.15 percent 24.15 percent 21.75 percent 27.13 percent

80. Determine risk of a portfolio of 50 % M and 50 % N from the information given in problem 81. (a) (b) (c) (d) (e)

19.5 percent 21.7 percent 17.8 percent 23.0 percent 25.4 percent

Ans: a 81. Two assets, 3 and 4, have the following risk and return inputs: E(r3) = 14% E(r4) = 22% Standard deviation (3) = 19% standard deviation (4) = 32% Correlation coefficient = -1 Investment Planning – 19Oct 07

12

Directions for questions 82 to 84: Use the information in the table below to answer the questions. Assume the risk less rate is the average of the five annual T-bill returns. ABC FUND'S TIME RETURNS % 19XO 19X1 19X2 19X3 19X4

XYZ FUND'S RETURNS %

15 -6 17 18 22

10 -2 13 9 11

S&P500's RETURNS %

T-BILL'S RETURNS %

11 -5 12 11 13

6 5 7 6 7

82. Determine the Sharpe performance index for the ABC Fund for the 5-year period. (a) (b) (c) (d) (e)

.71 .68 .91 1.05 1.10

83. Determine the Sharpe performance index for the XYZ Fund for the 5-year period. (a) (b) (c) (d) (e)

.48 .38 .78 .92 1.05

84. Determine the Jensen performance measure (alpha) for the ABC Fund over the 5-year period. (a) (b) (c) (d) (e)

3.72 2.69 1.76 2.01 3.76

85. Duration for a zero coupon bond is less than its term to maturity. a) True b) False 86. Longer-term bonds are almost always more volatile in terms of price than short-term bonds for a given change in interest rates. a) True b) False 87. Bond price volatility is directly related to the bond's coupon. a) True b) False 88. Duration for a coupon-paying bond is always less than its term to maturity. a) True b) False 89. For any given maturity, bond price movements that result from an equal absolute decrease or increase in the yield-to-maturity are symmetrical. Investment Planning – 19Oct 07

13

a) True b) False 90. There is a direct relationship between a bond's coupon and duration. a) True b) False 91. As a bond's YTM increases, if other things are held constant, its duration decreases. a) True b) False 92. When a bond is selling at a discount, its YTM exceeds the coupon rate. a) True b) False 93. When a bond's YTM equals its coupon rate, the bond's price is less than par value. a) True b) False 94. 10 percent semiannual bond with a YTM of 12 percent and 10 years to maturity has a price equal to (a) (b) (c) (d) (e)

Rs.1,051.65 Rs.1,159.88 Rs.885.30 Rs.888.89 Rs.955.41

95. The price of the bond in above Problem after 2 years, assuming everything else stays the same, is (Hint:There will be 8 years until maturity.) (a) (b) (c) (d) (e)

Rs.1,130.55 Rs.935 Rs.757 Rs.868 Rs.898.94

96. A bond's duration measures which one of the following? (a) (b) (c) (d) (e)

The time structure of a bond's cash flows The bond's interest-rate risk Both a and b above The default risk of the bond issue None of the above

97. If the market rate of interest falls, a coupon-paying bond will (a) (b) (c) (d) (e)

Decrease in value Experience a decrease in duration Experience an increase in duration None of the above Both a and b above

98. A bond's reinvestment rate risk: (a) Refers to the problem of being able to purchase another bond with the same or higher YTM when the existing bond matures or is called (b) Is the risk of not being able to reinvest the coupons of a bond at the bond's YTM (c) Is the same as marketability risk (d) Both a and b 14 Investment Planning – 19Oct 07

(e) None of the above 99.14 If you expect a large decline in interest rates, which of the following investments should you choose? (a) (b) (c) (d) (e)

Money market fund Low-coupon short-term bond High-coupon short-term bond Long-term zero coupon bond Short-term zero coupon bond

100. Bonds with higher coupons, other things being the same, (a) (b) (c) (d) (e)

Have more interest-rate risk than bonds with smaller coupons Have less interest-rate risk than bonds with smaller coupons Have higher duration than smaller-coupon bonds Have lower duration than smaller-coupon bonds Both b and d

Source: Kaplan Investment Planning Investments - Options (moderate) 101. A client with a large, well-diversified common stock portfolio expresses concern about a possible market decline. However, he/she does not want to incur the cost of selling a portion of their holdings nor the risk of mistiming the market. A possible strategy for him/her would be A. B. C. D. E.

Buy an index call option. Sell an index call option. Buy an index put option. Sell an index put option. He cannot protect against the decline with these options.

Solution: The client will benefit if the market increases. Her portfolio, because it is diversified, should be highly correlated and move with the market. An index option also moves with the market and, therefore, would be a good hedge vehicle. A put should be used, because it will increase in value if the market should decline, thereby offsetting any losses on the portfolio. 102. The Performance Fund had returns of 19% over the evaluation period and the benchmark portfolio yielded a return of 17% over the same period. Over the evaluation period, the standard deviation of returns from the Fund was 23% and the standard deviation of returns from the benchmark portfolio was 21%. Assuming a risk-free rate of return of 8%, which one of the following is the calculation of the Sharpe index of performance for the fund over the evaluation period? A. B. C. D. E.

.3913. .4286. .4783. .5238. .5870.

Solution: Realized Return - Risk Free Return Sharp Index = Standard Deviation of Portfolio (0.19 – 0.08). 0.47826 = 0.23. Answer is C Investment Planning – 19Oct 07

15

Investments - Standard Deviation of Portfolio (moderate) 103. The standard deviation of the returns of a portfolio of securities will be ____________ the weighted average of the standard deviation of returns of the individual component securities. A. B. C. D. E.

Equal to. Less than. Greater than. Less than or equal to (depending upon the correlation between securities). Less than, equal to, or greater than (depending upon the correlation between securities).

Solution: Answer is option D, unless the correlation coefficient bet ween the stocks is equal to one, the standard deviation for the portfolio will be lower than the weighted average standard deviation for the portfolio. Investments - Stock Valuation (moderate) 104.According to fundamental analysis, which phrase best defines the intrinsic value of a share of common stock? A. B. C. D. E.

The par value of the common stock. The book value of the common stock. The liquidating value of the firm on a per share basis. The stock's current price in an inefficient market. The discounted value of all future dividends.

Solution: Answer is option E; the intrinsic value of a share of common stock is equal to the discounted present value of its cash flows.

Investments - Municipal Bonds (easy) 105. Municipal bonds that are backed by the income from specific projects are known as: A. B. C. D. E.

Income bonds. Revenue bonds. General obligation bonds. Debenture bonds. Project bonds.

Solution: Answer is option B. General obligation bonds are backed by the full taxing authority of the municipality, whereas revenue bonds are only repaid from revenues of a particular project. 106. A call option with a strike price of 110 is selling for 3½ when the market price of the underlying stock is 108. The intrinsic value of the call is: A. B. C. D. E.

0. 1½. 2. 3½. (2).

Solution: The option is out of the money; therefore the intrinsic value is zero. The premium of an option does not affect the intrinsic value of the option, hence (a) Investments – Arbitrage Pricing Theory (easy) 107. In contrast to the Capital Asset Pricing Model, the Arbitrage Pricing Theory (APT): A. B. C. D.

Is usually a multi-factor model. Is primarily using by arbitrageurs to profit from imperfections in security markets. Assumes a market portfolio. Is a useful technical indicator.

Investment Planning – 19Oct 07

16

Solution: Arbitrage Pricing Theory uses multiple regression (many factors) to determine a model or formula that has numerous factors. This model is then used to determine the value of a security. The CAPM is based on the single factor of Beta, which measures the level of systematic risk within a portfolio. Investments - Options (moderate) 108. With the same Rupee investment, which of the following strategies can cause the investor to experience the greatest loss? A. B. C. D. E.

Selling a naked put option. Selling a naked call option. Writing a covered call. Buying a call option. Buying the underlying security.

Solution: A call option has unlimited price potential which means that writing a call without the stock as a hedge will provide the greatest loss potential. Investments - Stock Valuation (difficult) 109. If the market risk premium were to increase, the value of common stock (everything else being equal) would: A. B. C. D. E.

Not change because this does not affect stock values. Increase in order to compensate the investor for increased risk. Increase due to higher risk-free rates. Decrease in order to compensate the investor for increased risk Decrease due to lower risk-free rates.

Solution: The market risk premium is the additional return for accepting the risk of the market. If the market premium increases with all else remaining the same, then the price of the stock would have to decrease. An increase in the market premium would also increase the discount rate used to value the stock. This higher discount rate will cause the present value of the cash flows to be smaller. Investments - Stock Valuation (difficult) 110.The current annual dividend of ABC Corporation is Rs.2.00 per share. Five years ago the dividend was Rs.1.36 per share. The firm expects dividends to grow in the future at the same compound annual rate as they grew during the past five years. The required rate of return on the firm's common stock is 12%. The expected return on the market portfolio is 14%. What is the value of a share of common stock of ABC Corporation using the constant dividend growth model? (Round to the nearest rupee) A. B. C. D. E.

Rs.11. Rs.17. Rs.25. Rs.36. Rs.54.

Solution: PV = Rs.1.36 FV = (Rs.2.00) N= 5 i = 8.02 d1 1.082 (2.00) 2.16 Value of common stock = == = Rs.54.00 RRR - g 0.12 – 0.08 0.04 RRR = Required Rate of Return g = growth rate Investments – Performance Measures (easy) 111. In computing portfolio performance, the Sharpe index uses , while the Treynor index uses for the risk measure. 1 Standard deviation. 2 Variance. 3 Correlation coefficient. Investment Planning – 19Oct 07

17

4 5

Coefficient of variation. Beta. A. B. C. D. E.

5; 1. 1; 3. 1; 4. 1; 5. 2; 5.

Solution: Answer is option D. Sharpe uses standard deviation and Treynor uses beta. Investments – Unit Investment Trusts (easy) 112.

Which of the following is/are characteristics of a municipal bond unit investment trust? 1 Additional securities are not added to the trust. 2 Shares may be sold at a premium or discount to net asset value. 3 Shares are normally traded on the open market (exchanges). 4 The portfolio is self-liquidating. A. B. C. D. E.

1 only. 1 and 4 only. 2 and 3 only. 2 and 4 only. 1, 2, 3, and 4.

Solution: Once capitalized, unit trusts do not accept additional funds. Unit trusts are self-liquidating. Units are generally traded directly with the fund at net asset value.

Investments – Bond Valuation (moderate) 113. A Rs.1,000 bond originally issued at par maturing in exactly 10 years bears a coupon rate of 8% compounded annually and a market price of Rs.,147.20. The indenture agreement provides that the bond may be called after five years at Rs.1,050. Which of the following statements is/are true? 1 The yield to maturity is 6%. 2 The yield to call is 5.45%. 3 The bond is currently selling at a premium, indicating that market interest rates have fallen since the issue date. 4 The yield to maturity is less than the yield to call. A. B. C. D. E.

1, 2, and 3 only. 1 and 3 only. 2 and 3 only. 4 only. 1, 3, and 4 only.

Solution: Answer is option A YTM YTC PV = (Rs.1,147.20) (Rs.1,147.20)PMT= 80 80N=10 5FV = Rs.1,000 Rs.1,050i = 6 5.45 The price of a bond is inversely related to changes in interest rates. Therefore, #1, #2, and #3 are all correct. Investments/ Retirement/Insurance – Regulatory Requirements (easy) 114. Which of the following statements about industry/regulatory relationships are true? 1 2

The insurance industry is primarily regulated by each of the 50 states. The majority of banks are subject to federal regulation by the Federal Reserve System and the Federal Deposit Insurance Corporation. 3 Pension plan funds are primarily subject to federal regulation. 4 The organized stock exchanges, such as the New York Stock Exchange, are primarily regulated by the individual states in which they are incorporated. 18 Investment Planning – 19Oct 07

A. B. C. D. E.

1, 2, and 3 only. 1 and 3 only. 2 and 4 only. 2 and 3 only. 1, 2, 3, and 4.

Solution: Answer is option A. Organized exchanges are regulated by the SEC, which is a Federal agency. All other statements are true. Investments – Risk (easy) 115. Which of the following are non-diversifiable risks? 1 2 3 4 5 6

Business risk. Management risk. Company or industry risk. Market risk. Interest rate risk. Purchasing power risk. A. 4, 5, and 6 only. B. 1, 2, and 3 only. C. 5, 6, and 2 only. D. 1, 3, and 4 only. E. 4, and 6 only.

Solution: Answer is option A. Non-diversifiable risks or systematic risks are those that affect the entire market, including market risk, interest rate risk, and purchasing power risk.

Investments – American Depository Receipts (easy) 116. American depository receipts (ADRs) are used to: 1 2 3 4

Finance foreign exports. Eliminate currency risk. Sell U.S. securities in overseas markets. Trade foreign securities in U.S. markets. A. 1 and 3 only. B. 1 and 4 only. C. 2 and 4 only. D. 4 only. E. 1, 2, and 4 only.

Solution: Answer is option D.ADRs are used to trade foreign securities in the U.S. ADRs are foreign shares denominated in U.S. dollars and do not eliminate currency risk. Investments – Bond Swaps (easy) 117. Which combination of the following statements about bond swaps is true? 1 2 3 4

A substitution swap is designed to take advantage of a perceived yield differential between bonds that are similar with respect to coupons, ratings, maturities, and industry. Rate anticipation swaps are based on forecasts of general interest rate changes. The yield pickup swap is designed to change the cash flow of the portfolio by exchanging similar bonds that have different coupon rates. The tax swap is made in order to substitute capital gains for current yield. A. B. C. D. E.

1, 2, and 3 only. 1 and 3 only. 2 and 4 only. 4 only. 1, 2, 3, and 4.

Investment Planning – 19Oct 07

19

Solution: Answer is option A. Statements #1, #2 and #3 are true. Statement #4 is false because tax swaps generally take advantage of capital losses by selling bonds, which have been devalued by increasing interest rates. Investments – Modern Portfolio Theory (moderate) 118.Modern "asset allocation" is based upon the model developed by Harry Markowitz. Which of the following statements is/are correctly identified with this model? 1 The risk, return and covariance of assets are important input variables in creating portfolios. 2 Negatively correlated assets are necessary to reduce the risk of portfolios. 3 In creating a portfolio, diversifying across asset types (e.g., stocks and bonds) is less effective than diversifying within an asset type. 4 The efficient frontier is relatively insensitive to the input variable. A. B. C. D. E.

1 and 2 only. 1, 2, and 3 only. 1 only. 2 and 4 only. 1, 2, and 4 only.

Solution: Answer is option C. Statement #1 is true. It is not necessary to have negatively correlated assets; it is only necessary to have assets that have a correlation less than positive one (+1); thus, statement #2 is false. Statement #3 is false, because diversifying across asset types is more, not less, effective than within an asset type. Statement #4 is false, because all the input variables in statement #1 help to create the efficient frontier.

Investments - Municipal Bond Insurance (easy) 119. Municipal bonds are frequently insured. One of the insurers is the: a. Federal Insurance Guarantee Corporation. b. Resolution Trust Corporation. c. Associated Municipal Bond Corporation. d. Municipal Insurance Group. e. Municipal Bond Insurance Association. Solution: Municipal Bond Insurance Association (MBIA) is the only choice that insures municipal bonds. Investments – Security Types (easy) 120. Which one of the following products is designed to provide both growth and income? A. Fixed premium annuity. B. Non-participating mortgage real estate investment trust (REIT). C. Aggressive growth mutual fund. D. Convertible bond. Solution: Convertible bonds generate current income from coupon payments and allow for growth through the stock conversion feature. Options “A” & “B” provide income only and Option “C” is designed for growth. Investments - Options (easy) 121.Jennifer is optimistic about the long-term growth of her Widget stock. However, the stock, currently priced at Rs.58, has made a sharp advance in the last week and she wants to lock in a minimum price in case the shares drop. What might Jennifer do? A. B. C. D.

Buy Rs.55 call options. Sell Rs.55 call options. Buy Rs.55 put options. Sell Rs.55 put options.

Solution: Answer is option C. The exercise price for a put is the price at which you can sell the stock. Thus, a price of Rs.55 will be assured if she buys a Rs.55 put. Investment Planning – 19Oct 07

20

Investments – Stock Valuation (moderate) 122.Company ABC is currently trading at Rs.35 and pays a dividend of Rs.2.30. Analysts project a dividend growth rate of 4%. Your client Tom requires a rate of 9% to meet his stated goal. Tom wants to know if he should purchase stock in Company ABC. A. B. C. D. E.

Yes, the stock is undervalued. No, the stock is overvalued. No, the required rate is higher than the projected growth rate. Yes, the required rate is higher than the expected rate. No, the required rate is lower than the expected rate.

d0 (1 + g)

Rs.2.30 (1.04)

Solution: V = = = Rs.47.84 (value using dividend growth model), k - g 0.09 - 0.04 Since the value of Rs.47.84 is greater than the current market price, the stock is underpriced in the market. Tom should purchase the stock. Investments – Security Types (easy) 123. Your client will only invest in securities backed by the full faith and credit of the US government. Which of the following would you consider for his portfolio? A. B. C. D. E.

Federal Home Loan Mortgage Association Debentures (“Freddie Macs”). Federal National Mortgage Association Debentures (“Fannie Maes”). Government National Mortgage Association Certificates (“Ginnie Maes”). Student Loan Marketing Association Notes (“Sallie Maes”). Tennessee Valley Authority (TVA) Securities.

Solution: Answer is option C. Ginnie Maes are the only securities listed that are backed by the full faith and credit of the U.S. Government. Investments – Investment Planning Process (moderate) 124.If the client needs to accumulate wealth but is risk-averse, which of the following is the most crucial action the planner needs to take to have the client achieve the goal of wealth accumulation? Advise investing the client‟s current assets A. In the products which will bring the highest return to the client regardless of risk. B. In products that produce high income for the client because fixed income products are generally safe. C. In diversified mutual funds because of the protection which diversity provides. D. After determining the client’s risk tolerance. F. In 100% cash equivalents in the portfolio because most software programs recommend this safe approach. Solution: Answer is option D. One of the first steps that must be taken in planning for wealth accumulation is to determine the risk tolerance of the investor. 125. Your client is contemplating the exchange of two parcels of investment land for two similar parcels. Given the following details of the proposed transactions, compute the amount of recognized gain and loss (if any) on both parcels if your client does the exchanges. Parcel A: ten acres of land acquired 15 years ago with a current basis of Rs.50,000. In exchange your client will receive eight acres of land (FMV Rs.80,000) and Rs.20,000 of cash. Parcel B: twenty acres of land acquired two years ago with a current basis of Rs.100,000. In exchange your client will receive twelve acres of land (FMV Rs.75,000) and Rs.10,000 of cash. Parcel A Parcel B Recognized Gain Recognized Loss A. Rs.20,000 Rs.0 B. Rs.20,000 Rs.10,000 C. Rs.50,000 Rs.10,000 D. Rs.20,000 Rs.15,000 E. Rs.50,000 Rs.15,000 Investment Planning – 19Oct 07

21

Solution: Answer is option A A: New Parcel Rs.80,000Cash Received Rs.20,000Adjusted Basis of Old Parcel (50,000) Gain Realized Rs.50,000 The client must recognize gain up to the boot received (cash) of Rs.20,000. B :New Parcel Rs.75,000Cash Received Rs.10,000Adjusted Basis at Old Parcel (100,000)Loss Realized (Rs.15,000) The client will not recognize loss in a like-kind exchange. The client will have a new asset worth Rs.75,000 with a basis of Rs.90,000 (Rs.100,000 - 10,000). Investments – Security Valuation (moderate) 126. Which of the following would result in the largest increase in the price of a diversified common stock mutual fund? A. B. C. D.

Unexpected inflation. Expected dividend increases. Unexpected corporate earnings growth. Expected increase in the prime interest rate.

Solution: Answer is option C. Of all the choices, only unexpected corporate earning growth would have a significant positive impact on the value of a stock. Under the Efficient Market Hypothesis, any expected changes would already be reflected in the price of the common stock. Unexpected increases in inflation would increase the discount rate and reduce the value of the fund.

Investments - Capital Market Line (moderate) 127. In analyzing the position of a portfolio in terms of risk/return on the capital market line (CML), superior performance exists if the fund's position is the CML, inferior performance exists if the fund's position is the CML, and equilibrium position exists if it is the CML. A. B. C. D. E.

Above; on; below. Above; below; on. Below; on; above. Below; above; on. On; above; below.

Solution: Answer is option B. Above the line would indicate a higher than expected return for the given risk level. On the line would indicate an expected return for the given risk level. Below the line would indicate lower than expected return for the given risk level. Investments – Fixed-Income Securities (easy) 128. Which one of the following best describes a debenture? A. B. C.

A long-term corporate promissory note. An investment in the debt of another corporate party. A long-term corporate debt obligation with a claim against securities rather than against physical assets. D. A corporate debt obligation that allows the holder to repurchase the security at specified dates before maturity. E. Unsecured corporate debt. Solution: Answer is option E. A debenture is an unsecured corporate debt. Investments - Immunization (difficult) 129. A client has a cash need at the end of seven years. Which of the following investments might initially immunize the portfolio? 1 A 9-year maturity coupon bond. 2 A 7-year maturity coupon Treasury note. Investment Planning – 19Oct 07

22

3

A series of Treasury bills. A. B. C. D. D.

1, 2, and 3. 1 only. 2 and 3. 2 only. 1 and 2.

Solution: Answer is option B. The goal of immunization is to match the investment time horizon with the duration of the portfolio. Since the duration of a coupon bond is less than its maturity, only the 9-year bond might immunize the portfolio. The other two choices will not immunize the portfolio. The best choice would be a 7-year zero-coupon bond; however, this choice is not available. Investments - Risk (moderate) 130. Which combination of the following statements about investment risk is correct? 1 Beta is a measure of systematic, non-diversifiable risk. 2 Rational investors will form portfolios and eliminate systematic risk. 3 Rational investors will form portfolios and eliminate unsystematic risk. 4 Systematic risk is the relevant risk for a well-diversified portfolio. 5 Beta captures all the risk inherent in an individual security. A. B. C. D. E.

1, 2, and 5. 1, 3, and 4. 2 and 5. 2, 3, and 4. 2 and 5.

Solution: Answer is option B. Systematic risk cannot be eliminated, thus statement #2 is false. Beta only measures systematic risk; statement #5 is false. All other statements are true.

Investments - Rupee Cost Averaging (moderate) 131. Which combination of the following statements is true regarding the investment strategy known as “Rupee-cost averaging”? 1 Invests the same Rupee amount each month over a period of time. 2 Purchases the same number of shares each month over a period of time. 3 Lowers average cost per share over a period of time (assuming share price fluctuations). 4 Invests the same Rupee amount each month to protect the investment from loss of capital. A. B. C. D. E.

1 and 2. 1 and 3. 2 and 3. 2 and 4. 1, 2, 3, and 4.

Solution: Answer is option B. Rupee cost averaging is when an equal rupee amount is invested periodically. This does not prevent capital losses but can lower the average cost per share due to periods of decline in the stock price. Investments - Bonds (Difficult) 132. Harry Ingram purchased ten listed bonds (Widget Corp 8.00s10/1/99) on June 24, 1995, at a market asked price of 95. His transaction cost from the trade was Rs.100. He paid his broker Rs.9,800 as a consequence of the trade. He asks you to explain the details of his purchase. You reply that: 1

His purchase cost included Rs.9,500 for the bonds, Rs.100 for broker commission and Rs.200 as mark-up by the trade specialist. 2 His broker will report Rs.400 on Form 1099-INT as 1995 taxable interest on these bonds. His taxable interest income from the bonds for 1995 was Rs.400. 3 His purchase cost included Rs.9,500 for the bonds, Rs.100 for broker commission and Rs.200 as accrued interest. 4 His broker reported Rs.400 on form 1099-INT as 1995 taxable interest on these bonds. His taxable interest income from the bonds for 1995 was Rs.200. 5 His purchase cost included Rs.9,500 for the bonds, Rs.100 for broker commission and Rs.200 23 Investment Planning – 19Oct 07

charged in error by the brokerage house. A. B. C. D. E.

3 and 4. 1 and 2. 1 and 4. 2 and 3. 2 and 5.

Solution: Answer is option A Rs.800 per year (annual coupon). Rs.400 per coupon payment (semi-annual coupon). Rs.200 represents accrued interest, because the bond was purchased at a date in the middle of the coupon payment period (not on an interest date). Therefore, the Rs.9,800 is comprised of the following: Rs.9,500 Bonds100 Commission200 Accrued interest Rs.9,800 Since he received a coupon payment of Rs.400, he will receive a Form 1099 for the same amount. However, only Rs.200 of the Rs.400 will be taxable since only Rs.200 has been earned. Rs.200 was accrued and prepaid.

Investments - Taxation of Securities (moderate) 133. Your client is designing an educational investment program for her eight year-old son. She expects to need the funds in about ten years when her AGI will be approximately Rs.45,000. She wants to invest at least part of the funds in tax-exempt securities. Identify which investment(s) would yield taxexempt interest on her federal return if the proceeds were used to finance her son‟s education. 1 Treasury bills. 2 EE bonds. 3 GNMA funds. 4 Zero coupon Treasury bonds. A. B. C. D. E.

3 and 4. 1, 3, and 4. 2 and 3. 2 and 4. 2 only.

Solution: Answer is option E. Of the choices, only EE bonds can generate Federal tax -exempt income. T-Bills and T-Bonds generate state tax-exempt income but not Federal tax-exempt income. GNMA funds are taxable at the state and Federal level. Directions for questions 134 and 135: Refer to the caselet below and answer the questions Smith invests in a limited partnership that requires an outlay of Rs.9,200 today. At the end of years 1 through 5, he will receive the after-tax cash flows shown below. The partnership will be liquidated at the end of the fifth year. Smith is in the 28% tax bracket. YEARS 0 1 2 3 4

CASH FLOWS (Rs.9,200) Rs.600 Rs.2,300 Rs.2,200 Rs.6,800

Investment Planning – 19Oct 07

CFo CF1 CF2 CF3 CF4 24

5

Rs.9,500

CF5

Fundamentals - IRR (moderate) 134. The after-tax IRR of this investment is: A. B. C. D. E.

17.41%. 19.20%. 24.18%. 28.00%. 33.58%.

Solution: Answer is option C CFo = (Rs.9,200) CF1 = Rs.600 CF2 = Rs.2,300 CF3 = Rs.2,200 CF4 = Rs.6,800 CF5 = Rs.9,500 IRR = 24.18%

Fundamentals - IRR (moderate) 135. Which of the following statements is/are correct? 1 The IRR is the discount rate, which equates the present value of an investment's expected costs to the present value of the expected cash inflows. 2 The IRR is 24.18% and the present value of the investment's expected cash flows is Rs.9,200. 3 The IRR is 24.18%. For Smith to actually realize this rate of return, the investment's cash flows will have to be reinvested at the IRR. 4 If the cost of capital for this investment is 9%, the investment should be rejected because its net present value will be negative. A. B. C. D. E.

2 and 4. 2 and 3. 1 only. 1, 2, and 3. 1 and 4.

Solution: Answer is option D. Statement #4 is false, because if the cost of capital is less than the IRR, then the project should be accepted (NPV > 0). Directions for questions 136: Refer to the caselet below and answer the questions The tax bracket and holdings of your client are as follows: •

Federal tax bracket = 33%

Investment*

Annual Income

June 30, Last Year Purchase Price

June 30, This Year Market Price

Money fund

Rs.6,500

Rs.100,000

Rs.100,000

11% T bonds

Rs.11,000

Rs.100,000

Rs.140,000

S&P Index fund

Rs.6,000

Rs.100,000

Rs.160,000

Investment Planning – 19Oct 07

25

Computer stock fund

Rs.3,000

Rs.100,000

Rs. 85,000

*There have been no capital gains distributions. th

th

136. During the 12 months from June 30 , last year, through June 30 , this year, the portfolio earned, in annual yield and before-tax appreciation, respectively: A. B. C. D.

5.5% and 17.5%. 5.5% and 21.3%. 6.6% and 17.5%. 6.6% and 21.3%.

Solution: Answer is option D Total annual income Rs.26,500 =

= 6.63% annual yield

FMV @ June 30 last year Rs.400,000 Change in value Rs.85,000

Investments – Duration (moderate) 137.Assuming that the duration of Bond A is 1.94 years, which of the following statements about the effect of a 1% decline in interest rates is true? A. B. C. D. E.

Bond C, having a longer duration than Bond A, would have a larger percent increase in price than Bond A. The percent change in price of a bond is independent of the duration of a bond. It is not possible to determine the percent change in price of Bond A versus Bond C because the duration of Bond C is not given. Bond A would have a greater percent change in price than Bond C because it has a shorter duration. The percent change in the price of Bonds A and C is equal since it is not affected by duration.

Solution: Answer is option A. The bond with the longest duration has the greatest interest rate risk. The greater the duration, the more effect a change in interest rates has on the bond‟s value. A zero coupon bond‟s duration equals its time to maturity. FMV @ June 30 last year = Rs.400,000 = 21.25% appreciation (Before tax) Investments – Bonds (easy – application) 138. The following set of newly issued debt instruments was purchased for a portfolio: Treasury bond Zero-coupon bond Corporate bond Municipal bond. The respective maturities of these investments are approximately equivalent. Which one of the investments in the preceding set would be subject to the greatest relative amount of price volatility if interest rates were to change quickly? A. B. C. D.

Treasury bond. Zero-coupon bond. Corporate bond. Municipal bond.

Investment Planning – 19Oct 07

26

Solution: Answer is option B. A zero-coupon bond‟s duration equals its maturity. Coupon bonds (the remaining options) have durations less than their maturities. Higher duration bonds are subject to more price volatility than lower duration bonds. Investments – Bonds (easy – application) 139. Which of the following best describes the investment characteristics of a high-quality long-term municipal bond? A. B. C. D.

High inflation risk; low default risk. Low inflation risk; high market risk. Low inflation risk; low default risk. High inflation risk; high market risk.

Solution: Answer is option A. A longer-term bond will be subject to more inflation risk. Since the quality of the bond is high, the level of default risk is low. Investments –Options (moderate) 140. Jasmine has a large paper profit in her Amalgamated Corporation shares, currently at 46. She is happy with the stock but realizes that a good thing cannot go on forever. If she is willing to sell at 50, what strategy could you recommend to her? A. B. C. D.

Buy Rs.50 call options. Sell Rs.50 call options. Buy Rs.50 put options. Sell Rs.50 put options.

Solution: Answer is option B. Selling a call option will allow her to generate income from the option premium with little risk since she does not expect the stock to continue to increase. If the stock does exceed Rs.50, she would be paid what she wants for the stock. Investments –Security Pricing Theories (easy) 141. “Stock prices adjust rapidly to the release of all new public information.” This statement is an expression of which one of the following ideas? A. Random walk hypothesis. B. Arbitrage pricing theory. C. Semi-strong form of the efficient market hypothesis. D. Technical analysis. Solution: Answer is option C. The statement in the question is referring to the definition of the semi strong form of the efficient market hypothesis. Investments – Bond Valuation (moderate) 142. Assuming that the current market yield for similar risk bonds is 8%, determine the discounted present value of a Rs.1,000 bond with a 7.5% coupon rate, which pays interest semiannually and matures in 17.5 years. A. B. C. D. E.

Rs.504.68. Rs.539.78. Rs.953.34. Rs.968.96. Rs.1,653.26.

Solution: Answer is option C N = 35 (17.5 x 2)i = 4 (8  2)PMTOA = Rs.37.50 (Rs.75  2) FV = Rs.1,000PV = (Rs.953.34) Investments – Performance Measurements (moderate) 143. Given the following diversified mutual fund performance data, which fund had the best riskadjusted performance if the risk-free rate of return is 5.7%? Fund

Average Annual Return

Investment Planning – 19Oct 07

Standard Deviation of

Standard Deviation of

Beta 27

A B C D

.0782 .1287 .1034 .0750

Annual Return .0760 .1575 .1874 .0810

Annual Return 0.950 1.250 0.857 0.300

A. Fund B because the annual return is highest. B. Fund A because the standard deviation is lowest. C. Fund C because the Sharpe ratio is lowest. D. Fund D because the Treynor ratio is highest. E. Fund A because the Treynor ratio is lowest. Solution: Answer is option D. Generally, investors evaluate performance of investments based on risk adjusted returns. Therefore, A and B must be wrong since they only address one aspect of the risk-return relationship. Treynor and Sharpe ratios are performance measures in which the higher the ratio, the better the risk adjusted return. No calculation is needed for this question. D is the only reasonable answer. It is the only one that indicates that a fund should be chosen, because the performance measure is high (not low). Investments – Immunization (moderate) 144. To immunize a bond portfolio over a specific investment horizon, an investor would do which of the following? A. B. C. D.

Match the maturity of each bond to the investment horizon. Match the duration of each bond to the investment horizon. Match the average weighted maturity of the portfolio to the investment horizon. Match the average weighted duration of the bond portfolio to the investment horizon.

Solution: Answer is option D. To immunize a bond portfolio, an investor should match the duration of the bond portfolio to the investment time horizon. Investments – Mortgage Backed Securities (easy) 145. Mortgage-backed securities may contain which of the following risks? 1 Purchasing power risk. 2 Interest rate risk. 3 Prepayment risk. A. B. C. D.

2 only. 1 and 2 only. 1 and 3 only. 1, 2, and 3.

Solution: Answer is option D. Mortgage–backed securities are subject to the same risk as bonds plus the risk of prepayment. Investments – Investment Planning Process (moderate) 146.You receive a phone call from an individual you have not spoken with previously. The caller is excited, just having heard that a new mutual fund is positioned to deliver large gains in the coming year. The caller wishes to purchase shares of the fund through you. Keeping in mind stages of the overall personal financial planning process, which of the following questions that addresses the first two stages of the financial planning process should you ask the caller? 1 What are your goals for this investment? 2 What other investments do you have? 3 What is your date of birth? 4 Do you want your dividends reinvested? A. B. C. D.

1 and 3 only. 2 and 4 only. 1, 2, and 3 only. 1, 2, and 4 only.

Solution: Answer is option C. Question #1 should always be asked. Question #2 is a reasonable Investment Planning – 19Oct 07

28

question, since it provides the planner with a concept of the client‟s investment experience leading to suitability. This type of analysis is a part of data gathering. Question #3 goes to suitability and should be asked, or the age should be determined relative to risk and time horizon. The question relates to the first two stages of data collection and goals. Reinvestment of dividends does not go to suitability and will be addressed after the investment decision is made. Below is the CFP Board of Examiners‟ clarification of the question: (1), (2), and (3) all relate to the second step of the financial planning process, namely, “Gathering client data and determining goals and expectations”. The first step is “Establishing the client-planner relationship”. Fundamentals – Regulatory Requirements (easy) 147. Which of the following are exemptions under the definition of “investment adviser”? 1 2 3 4

Banks that are not investment companies. Accountants or lawyers whose investment advice is “solely incidental” to the practice of their profession. Persons whose advice relates only to securities issued or guaranteed by the U.S. government. Publishers of financial publications that have regular and general circulation. A. B. C. D.

1 and 3 only. 2 and 4 only. 1, 2, and 4 only. 1, 2, 3, and 4.

Solution: Answer is option D. All are exempt under the definition of “investment advisor”.

Investments – Duration (easy) 148. The duration of a bond is a function of its: 1 2 3 4

Current price. Time to maturity. Yield to maturity. Coupon rate. A. B. C. D. E.

1 and 3 only. 2 and 3 only. 2 and 4 only. 1, 2, and 3 only. 1, 2, 3, and 4.

Solution: Answer is option E. Duration is a present value, time weighted measure of payback. All of the factors listed are important components in determining duration. Investments – Bonds (easy) 149. You are faced with the following alternative fixed income investments. A. A U.S. Treasury bond with an 11.625% coupon, due in 2004 with a price of Rs.142.50 and a yield to maturity of 6.3%. B. A U.S. Treasury strip bond (zero coupon) due in 2004 with a price of Rs.46.75 and a yield to maturity of 6.25%. C. A corporate B-rated bond with a 9.75% coupon, due in 2004 with a price of Rs.104.75 and a yield to maturity of 8.79%. 1 2 3

Which of these bonds has the greatest reinvestment rate risk? Which of these bonds has the greatest interest rate risk? Which of these bonds has the longest duration?

Solution: Answer is option is A. Reinvestment rate risk is the risk that cash flows that occur during the Investment Planning – 19Oct 07

29

holding period of the investment will not be able to be reinvested at a rate of return at least as great as the internal rate of return (or yield to maturity) of the investment. Since Bond A has the highest coupon payment (and, therefore, the highest cash flow during the holding period), it has the greatest amount of reinvestment rate risk. 150.

Investment A costs Rs.10,000,000 and offers a single cash inflow of Rs.13,000,000 after 1 year. Investment B costs Rs.1,000,000 and will be worth Rs.2,000,000 at the end of the year. The appropriate discount rate or required rate of return is 10% compounded annually. Match the investment(s) listed below with the corresponding financial information in the items that follow. A. Investment A. B. Investment B. C. Both A and B. D. Neither A nor B.

1 2 3

The net present value (NPV) is Rs.818,182 and the internal rate of return is 30%. The NPV is Rs.818,182 and the internal rate of return is 100%. The NPV is Rs.1,818,182 and the internal rate of return is 30%.

Solution: Answer is option D. Net Present Value (NPV) is defined as the difference between the discounted inflows and the initial outflow. For example, if the discounted cash inflows were Rs.10 and the initial outflow was Rs.8, then the NPV would be Rs.2.

Net Present Value Investment A

Investment B

FV

=

Rs.13,000,000.00

Rs.2,000,000.00

N

=

1

1

i (discount rate)

=

10%

10%

PV Less Initial Outflow

=

Rs.11,818,181.82

Rs.1,818,181.82

=

(10,000,000.00)

(1,000,000.00)

NPV

=

Rs.1,818,181.82

Rs.818,181.82

Internal Rate of Return Investment A

Investment B

FV

=

Rs.13,000,000.00

Rs.2,000,000.00

N

=

1

1

PMT

=

Rs.0

Rs.0

PV

=

(Rs.10,000,000.00)

(Rs.1,000,000.00)

i

=

30%

100%

Investments – Securities (moderate) 151. Which of the following is the correct justification for use of an investment in a client‟s portfolio? A. FNMA securities because they are backed by the full faith and credit of the U.S. government. Investment Planning – 19Oct 07

30

B. Call option purchases because they are a low-cost method of hedging a portfolio. C. Blue chip common stocks because they provide a hedge against inflation. D. Aggressive growth stocks because they perform better during economic contractions. Solution: Answer is option C; blue chip common stocks provide a good hedge against inflation, because they typically provide a return that significantly outpaces inflation. A is incorrect because the government does not back FNMA securities. B is incorrect because Call options purchased by a client will not help protect a client from a downturn in the market. D is incorrect because aggressive growth stocks generally perform worse during economic contractions than less aggressive investments. Investments – Security Valuation (easy) 152. The Zeta Corporation‟s current dividend is Rs.3.85. If future dividends are expected to grow at 4% forever, which of the following amounts should Zeta stock sell for if the required rate of return on the stock is 14%? A. Rs.28.57. B. Rs.38.50. C. Rs.40.04. D. Rs.41.60. Solution: Answer is option C. This is a question regarding the constant dividend growth model for determining the value of a stock. The following formula is used for the constant dividend growth model: D1P0 = where: k-g

P0 = Price for the security.D1 = The dividend paid at period 1. k = The investor‟s required rate of return. G = The growth rate of the dividends. Therefore, the value of the stock is Rs.40.04, calculated as follows: (Rs.3.85)(1.04) P0 = = Rs.40.04 .14 - .04

Investments - Bonds (moderate) 153.Treasury zero-coupon bonds are particularly suited to which of the following types of accounts? A. IRA. B. Trust. C. Corporate. D. Joint. Solution: Answer is option A. Treasury zero-coupon bonds do not pay interest to the investor. However, the accrued interest is taxable each year. Therefore, this type of investment is best suited for a taxadvantaged account. Investments - Securities (moderate) 154. A client has a growth objective but requires a large percentage of the return to be tax efficient. Which of the following products would be most appropriate for this client? A. B. C. D.

Nonleveraged equipment leasing. Balanced mutual fund. Preferred stock mutual fund. Stock index fund.

Solution: Answer is option D.A stock index fund is a mutual fund that mirrors a stock index. Typically, the fund pays little or no dividends, and is therefore tax efficient. A is incorrect, nonleveraged equipment leasing investments have more of an income objective than a growth objective and are not tax efficient because the income is taxed each year. B is incorrect because A balanced mutual fund usually invests a large percentage of the fund assets in fixed-income securities. C is incorrect because preferred stocks often pay a large dividend and usually have an income objective.

Source INV Plng 24 Q Word Doc Investment Planning – 19Oct 07

31

155. Rani invested Rs.10,000 in a fixed deposit. If the interest is compounded monthly at an annual rate of 4%, what would be the amount that Ali would receive in five years time? (use 2 decimal places) a. b. c. d.

12,158.65 12,185.65 12,188.65 12.187.65

156. Rekha has borrowed Rs.2,500 at 2% annual interest rate compounded quarterly. What is the amount she has to repay after five years? e. f. g. h.

2,762.24 2,763.24 2,769.24 2,768.28

157. Avni wants to accumulate Rs.15,000 in three years time for a one-month USA trip upon her graduation. Assuming she can get 6% annual return compounded semi-annually from her investment, how much must she invest today to achieve her goal? a. b. c. d.

12561.26 12562.11 12562.26 12564.26

158. Chetan borrowed Rs.5,000 five years ago. Assuming that he has to repay Rs.6,500 now, how much interest rate was he charged? 5.28 5.39 5.93 5.82 159. A bond of face value of Rs. 1000 has a coupon of 7.5% and is compounded quarterly duration 4 yrs. similar bond in market yield 8% what is pv of the bond? a) 982.03 b) 983.03 c) 984.03 d) 985.03 160. Mona invests Rs. 21000 for 3 yrs @ 6 %p.a. compounded annually for first 1 yr and quarterly next 2 years. What would be her maturity benefit?

for

25076.35 25075.72 18951.71 20277.68 161 A bond of face value of Rs. 1000 has a coupon of 7.5% and is compounded semi annually duration 4.5 yrs. similar bond in market yield 8% what is PV of the bond? 981.41 982.42 983.25 980.25 162. Mahesh invests Rs. 18000 for 4 yrs @ 8 % p.a. compounded quarterly for first 2 yrs and annually for next 2 years. What would be his maturity benefit? 24599.22 24566.33 25501.33 Investment Planning – 19Oct 07

32

26502.23 163. A bond of face value of Rs. 1000 has a coupon of 8.5% and is compounded annually, duration 12 yrs., similar bond in market yield 9% what is PV of the bond? 963.20 964.20 964.80 965.80 164. Mary invests Rs. 15000 for 3 yrs @ 8 % p.a. compounded annually for first 1 yr and half yearly for next 2 years. What would be her maturity benefit? 18524.21 18951.71 19850.71 19849.21 165.Mr. Singh buys 200 convertible debentures of TISCO at Rs. 200 each. 50% of the value of these debentures is converted into one share of Rs. 80 each after 4 years. Mr. Singh exercises his options after 4 yrs and receives 100 shares. Compute cost of acquisition of each share? a) b) c) d)

200 250 275 300

166. Meena invests Rs. 16000 for 4 yrs @ 6 % p.a. compounded annually for first 1 yr and quarterly for next 3 years. What would be her maturity benefit? a) b) c) d)

20277.68 20278.22 22285.25 20275.78

167. Mr. Satish buys 300 convertible debentures of ABB at Rs. 300 each. 50% of the value of these debentures is converted into one share of Rs. 80 each after 5 years. Mr. Singh exercises his options after 5 yrs and receives 120 shares. Compute cost of acquisition of each share? a) b) c) d)

350 360 375 380

168. A bond of face value of Rs. 1000 has a coupon of 7.5% and is compounded semi annually, duration 17.5 yrs., similar bond in market yield 8% what is PV of the bond? a) b) c) d)

956.34 953.34 1074.43 987.60

169. Miss Savi buys 250 convertible debentures of Reliance Ind. at Rs. 300 each. 60% of the value of these debentures is converted into one share of Rs. 80 each after 7 years. Miss. Savi exercises his options after 7 yrs and receives 100 shares. Compute cost of acquisition of each share? a) b) c) d)

400 420 425 450

170. A bond of face value of Rs. 1000 has a coupon of 6.5% and is compounded semi annually, duration 13.5 yrs. similar bond in market yield 7% what is PV of the bond? Investment Planning – 19Oct 07

33

a) b) c) d)

957.79 985.79 956.79 958.79

171. Miss Seema buys 250 convertible debentures of Asian hotels at Rs. 300 each. 60% of the value of these debentures is converted into one share of Rs. 100 each after 7 years. Miss. Seema exercises his options after 7 yrs and receives 150 shares. Compute cost of acquisition of each share? a) b) c) d)

225 250 275 300

172. A bond yield 10% annually with interest paid semi- annually. Duration is 3 years. The yield on bond is 12%. Find current price. a) b) c) d)

1000.83 1050.43 986.36 950.83

173. Mrs. Shikha buys 250 convertible debentures of ACC. at Rs. 300 each. 60% of the value of these debentures is converted into one share of Rs. 50 each after 8 years. Mrs. Shikha exercises his options after 8 yrs and receives 200 shares. Compute cost of acquisition of each share? 200 225 250 275

174. A bond of face value of Rs. 1000 has a coupon of 7.5% and is compounded quarterly, duration 16 yrs. similar bond in market yield 8.5% what is PV of the bond? 912.98 912.38 913.28 914.88 175. Ravi invests Rs.15000 @12% compounded monthly .What will be the value of his investment after 6 years? 31706.49 30706.49 30705.29 30704.29 176. Mr. Sushil buys 300 convertible debentures of Telco at Rs. 250 each. 40% of the value of these debentures is converted into one share of Rs. 50 each after 6 years. Mr. Sushil exercises his options after 6 yrs and receives 80 shares. Compute cost of acquisition of each share? 375 380 385 390 177. A bond of face value of Rs. 1000 has a coupon of 7.5% and is compounded semi annually duration 5 yrs. similar bond in market yield 8% what is PV of the bond? 978.62 Investment Planning – 19Oct 07

34

978.42 978.52 979.72 178. Reshma invests Rs. 5000 @8% compounded monthly .What will be the value of his investment after 4 years? 6877.33 6876.33 6878.33 6876.88 Source: C:\WINDOWS\TEMP\quiz-apr-inv1.html 179. The best method of valuing a share is: a) b) c) d)

Book value based on net tangible assets. Liquidation value based on the proceeds of liquidation of the company. Present value of all the dividends to be received from holding that share. Apply the P / E ratio to expected earnings per share.

180. If a bond is selling at a premium ______________________________ a) b) c) d)

It is an attractive investment Its realised compound yield will be less than the yield to maturity Its coupon rate is below market rate Its current yield is lower than the coupon rate

181. Which of the following is NOT a characteristic of a balance fund? a) b) c) d)

It provides both growth and income objectives It is less risky than growth funds It is more risky than income funds It must invest in both equity and bonds in equal amount

182. Purohit has just bought a house for Rs.8,00,000. He wants to sell it in 20 years time for Rs. 20,00,000. His friend who is a real estate agent estimates that the house will appreciate in value by 6% for the first 8 years, 5% for the next 8 years and 4.50% for the last four years. Will will be the value of the house after 20 years and will Purohit be able to realise his desired value ? a) b) c) d)

21,85,489, Yes 22,46,552, Yes 19,54,413, No 17,45,387, No

183. Disclosure statements to prospective clients include all of the following except: a) b) c) d)

Performance record of other clients The method of remuneration, fees and commissions Access to internal and external complaint handling mechanism Disclosure of any conflict of interest

184. Which of the following costs best describes the cost of foregone income that results from making an economic decision to use funds to purchase a piece of equipment? a) b) c) d)

Cost of Capital Fixed Cost Marginal Cost Opportunity Cost

185. A mutual fund that invests in Indian Equities, foreign equities, Indian Corporate Bonds, Indian Government Gilts is subject to the following risks? Investment Planning – 19Oct 07

35

1.Business Risk, 2.Default Risk, 3. Systematic Risk, 4. Interest Rate Risk. a) b) c) d)

1 & 3 only 1,3 & 4 only 3 & 4 only 1,2,3 & 4

186. Which of the following statements concerning technical stock market indicators is/are correct? 1. The stock market is considered strong when the volume of the market is increasing in a rising market. 2. The market's direction will change when the percent of odd-lot short sales significantly increases or decreases. 3. Prices crossing the moving average line would be an indication of the change in the market. a) b) c) d)

1 only 1 & 2 only 2 & 3 only 1,2 & 3

187. If a new issue was offered to the public at 15 times earnings but the market was pricing similar shares at 19 times, this would be _____________. a) b) c) d)

Appalling proposition to the investor The investor cannot take a position An example of low gearing Bargain not to be missed

188. Mr. X buys 50 TISCO October Rs. 350 call options for Rs. 15. The current share price is Rs. 345. The break-even share price, ignoring transaction costs is Rs.________. a) b) c) d)

350 360 365 None of the above

189. Investor complaints relating to the following Capital Market issues will not be entertained by SEBI: a) A company declaring no dividend on equity for the fourth consecutive year. b) A company has declared dividend but not paid the same after six months of declaration. c) A company not paying the redemption proceeds on debentures issued by the company, one year after maturity date. d) None of the above cases. 190. If a bond is selling at a premium: a) b) c) d)

It is an attractive investment Its coupon rate is below market rate Its current yields is lower than the coupon rate Its realized compound yield will be less than the yields to maturity

191. In ranking portfolio performance, which measure of risk does the Treynor Index use? a) standard deviation b) variance c) beta d) alpha Investment Planning – 19Oct 07

36

Solution: By definition, the Treynor Index uses beta as its risk measure. 192. A portfolio manager can hedge a share portfolio by_______. a) b) c) d)

Buying call options Selling call options Buying index options Selling put options

193. The trust deed lays down the terms and conditions under which the unit holders money is to be invested. Specifically, it details: a) The unit trust scheme b) The types of authorized investments c) All fees and charges d) All of the above 194. Beta is a measure of: a) b) c) d)

Geometric average return Holding period return Systematic risk Unsystematic risk

195. If the current share price is S and the set exercise price is X, the intrinsic value of the call option is_______. a) b) c) d)

Max (O, S-X) Max (O, X-S) Min (O, S-X) Min (O, X-S)

196. The CAPM is a model that: a) b) c) d)

Determines the geometric return of a security. Determines time-weighted return Explain return in terms of risk. Explains systematic risk

197. GE is an AAA rated issuer of Corporate Bonds in the International Debt markets. The issue price of a typical GE corporate bond is affected by all the following EXCEPT the _______. a) b) c) d)

Face value, coupon rate, and maturity of the bond. Firms required return on debt. Percentage of debt in the firm's capital structure. Required return on the firm's competitors' bonds.

198. Public Issue through the book building process is better than I.P.O at fixed price because_______. a) High fixed price will result in under subscription leading to loss to the investor. b) It helps the issuer to ascertain the exact price at which the investor is willing to subscribe. c) Low fixed price will result in over subscription leading to loss to the issuer. d) All of the above 199. A client purchased a mutual fund with a Rs 10, 000 lump-sum amount four years ago. During the four years, Rs 4, 000 of dividends was reinvested. Today the shares are valued at Rs 20,000 (including any shares purchased with dividends). If the client sells shares equal to Rs13, 000, which statement (s) is/are correct? 1. The taxable gain can be based on an average cost per share. Investment Planning – 19Oct 07

37

a) b) c) d)

2. The client can choose which shares to sell, thereby controlling the taxable gain. 3. To minimize the taxable gain today; the client would sell shares with the higher cost basis. 4. The client will not have a gain as long as he/she sells less than what he/she invested. 1, 2, and 3 only. 1 and 3 only. 2 and 4 only. only.

200. The best method of valuing a share is: a) b) c) d)

Book value based on net tangible assets. Liquidation value based on the proceeds of liquidation of the company. Present value of all the dividends to be received from holding that share. Apply the P / E ratio to expected earnings per share.

201. Which combination of the following statements about investment risk is correct? 1. Beta is a measure of systematic, non-diversifiable risk. 2. Rational investors will form portfolios and eliminate systematic risk. 3. Rational investors will form portfolios and eliminate unsystematic risk. 4. Systematic risk is the relevant risk for a well-diversified portfolio. 5. Beta captures all the risk inherent in an individual security. A. B. C. D. E.

1, 2, and 5. 1, 3, and 4. 2 and 5. 2, 3, and 4. 2 and 5.

202. Which of the following asset allocations would you recommend to a 60 year old retiree who depends on his investments for monthly income? Fixed Deposits: 60% Properties: 30% Equities: 10% Fixed Deposits: 40% Properties: 30% Equities: 30% Fixed Deposits: 20% Properties: 40% Equities: 40% Fixed Deposits: 10% Properties: 40% Equities: 50% Solution: A retiree at 60 has little appetite for risk as he no longer has any earning power. Further, he needs certainty of income. But a small amount of equity is still recommended as a hedge against inflation. If not, if he lives until say 80, he will experience a drop in his standard of living. 203. The following are the long-term credit ratings given by CRISIL, Which is the lowest investment grade category? AAA AAA+ BBB+ BBB204. Rank the following funds in the order of increasing risk: I Growth Fund, a) b) c) d)

II Balanced Fund,

III Bond Fund ,

IV Small Cap Fund

III, IV, II, I III, II, IV, I III, II, I, IV III, IV, I, II

Investment Planning – 19Oct 07

38

Solution: It is generally accepted that the order is Bond Fund, Balanced Fund, Growth Fund and Small Cap Fund. A small cap fund is more risky than a growth fund. Earnings of smaller companies are more exposed to the vagaries of the economic cycle. 205. The Fund Manager for a particular scheme is an employee of the _______ a) b) c) d)

Sponsor AMC Trustee Custodian

206. Capital allocation forms a critical part of _____________ a) b) c) d)

Cash Flow Planning Investment Planning Will Planning Insurance planning

207. Implied volatility is a term commonly used in the __________ a) b) c) d)

Derivatives market Real Estate market Fixed Income Market Stockmarket

208. Studies show that the Indian middle class has an excess of ______ as investments. a) b) c) d)

Fixed Deposits Mutual Funds Equity Shares Real Estate

209. Only those equity transactions where ______ is paid are eligible for zero long term capital gains tax a) b) c) d)

Service tax Securities Transaction Tax Brokerage Depository Participant Charges

210. ______________ funds have the highest expense ratio a) b) c) d)

Customised Novated Standardised Without counterparty risk

211. Purchasing a Futures Contract __________ a) b) c) d)

Confers a right but no obligation on the buyer Confers an obligation but no right to the buyer Enables an investor to take a leveraged position Protects the buyer against downside risk

212. ______________ equity funds are appropriate for an Indian investor with a penchant for global diversification a) b) c) d)

Diversified Sector International Global

Investment Planning – 19Oct 07

39

213. Your liquid fund investment is treated as a ____________ asset a) b) c) d)

Intangible Investment Near-Cash None of the above

214. Which of the following factors will result in an increase in the duration of corporate bond.    

I an increase in the number of years to maturity. II a decrease in the coupon rate III change from annual to semiannual coupon payment. IV change from annual coupon to zero coupon bond a) b) c) d)

I, II & III only I, II and IV only I, III & IV only II, III & IV only

Answer: A change in the coupon payment from annual to semiannual will result in the coupons becoming more significant compared to the final payment on a weighted average basis and decreases the duration of a bond . All other factors will increase the duration of a bond. 215. A lump sum investment option in a mutual fund will be most warding in __________________ a) b) c) d)

A secular uptrend A secular downtrend Volatile markets A sideways market

216. Tata Power is raising funds through a bond issuance to fund a new power plant at Noida, UP. They are issuing Two Year maturity, Zero-coupon bond with face value of Rs 1000 and yield of 4%. What price would you pay for this Tata Power Zero-coupon bond today? a) b) c) d)

Rs. 920.00. Rs. 924.56. Rs. 925.95. Rs. 960.00.

217. Equal amount of investment is made in portfolio consisting of securities X and Y. Standard deviation of X is 12.43%. ; Standard deviation of Y is 16.54%. ; Correlation coefficient is 0.82. ; The interactive risk of the portfolio, measured by covariance is a) b) c) d)

145.64 156.22 168.59 172.56

218.The risk free return of Security A is 8%. In addition to it, you expect that the return on market would be 14%. The expected return of Security A with beta of 0.70 is ________. a) b) c) d)

12.2%. 15.4%. 17.8%. 18.2%.

219. The debt – asset ratio is a useful tool to primarily measure ______________ Liquidity Investment Planning – 19Oct 07

40

Solvency Savings potential Debt servicing capacity 220. Portfolio A had a return of 12% in the previous year, while the market had an average return of 10%. The standard deviation of the portfolio was calculated to be 20%, while the standard deviation of the market was 15% over the same time period. If the correlation between the portfolio and the market is 0.8, what is the Beta of the portfolio A? a) b) c) d)

0.94 1.07 1.31 1.91

Solution: Beta=[(S.D of security / S.D of Market) *Coefficient of correlation] 221. The following are statements made concerning contracts of insurance. Identify the statement/s that is/are correct. I. For life insurance contracts, misstatement of the insured‟s age constitutes avoidable misrepresentation. II. An innocent misrepresentation by an applicant for insurance constitutes fraud. A. I only B. II only C. I and II D. Neither I nor II

222. Which of the following statements concerning the movement in foreign exchange rates are true, assuming that all other factors remain unchanged?    

I The exchange rate will appreciate with an increase in domestic interest rates II Increasing international reserves leads to a stronger exchange rate III Strengthening the exchange rate leads to a temporary increase in the competitiveness of exports IV Lower domestic inflation leads to a weakened exchange rate a) b) c) d)

I & II only II & III only III & IV only I & IV only

Solution: III A strengthening exchange rate will lead to a drop in competitiveness of a countries exports – statement in false. IV Statement in false. Lower domestic inflation will lead a strengthening exchange rate. The other statements are true. 223. A corporation proposes to issue a 7-year bond with a coupon rate of 8.0%. The prevailing yield to maturity of bonds with similar risk and term is 10.0%.The bond will sell at ____________ to its face value. a) b) c) d)

a premium a discount par a predetermined price

Solution: Given that the coupon rate is less than the yield to maturity of similar instruments, the bond will sell at a discount. 41 Investment Planning – 19Oct 07

224. Calculate the net price earnings ratio (PER) of the stock of Company A, with the following information:    

Price = Rs.5.00 Profit before tax = Rs.75.0 million Profit after tax = Rs.54.0 milllion Paid up Capital = Rs.100 million at par value of Rs.0.50 per share

a) b) c) d)

6.7 9.3 13.3 18.5

Solution: The calculations are as follows: EPS = 54/200 = Rs.0.27, PER = 5.00/0.27 = 18.5 225. Mr A‟s stock market portfolio consistently outperformed the BSE Sensex based on tips he received from a friend working in a corporate finance department. Which of the following is his success a violation of? a) b) c) d)

weak form of market efficiency semi-strong form of market efficiency strong form of market efficiency total market efficiency

Solution: The strong form of market efficiency maintains that prices contain all information, private and public. The fact that Mr A was able to beat the BSE is a violation of the strong form of market efficiency

226. Which of the following have a negative impact on stock prices?    

I increase in risk premiums II increase in dividend growth rate III increase in the discount rate IV increase in interest rates a) b) c) d)

I II & III only I II & IV only I, III & IV only II, III & IV only

Solution: The only answer that does not contain II. An increase in the dividend growth rate leads to an increase in the stock price 227. What is the expected return of an investment with the following distribution of possible outcomes? RETURN (%) 7 12 20 5

Probability 20% 30% 15% 35% Total 100%

a) 25.00 b) 11.00 c) 9.75 Investment Planning – 19Oct 07

42

d) 9.50 Solution: (7x0.2) + (12x0.3) + (20x0.15) + (5x0.35) = 9.75 228. Stock A and stock B are positively correlated with a correlation coefficient of 0.75. When stock A moves up by 12%, how will stock B perform? a) b) c) d)

Stock B will move up by 12% Stock B will move down by 12% Stock B will move up by 9% Stock B will move down by 9.0%

Solution: By definition, as the stocks are positively correlated, they move in the same direction but Stock B will move three quarter as much as Stock A. 229. Which of the following statements regarding investment risk are true?  I Beta captures all the inherent risks in an individual security  II Unsystematic risk is reduced in a portfolio because securities are not perfectly correlated  III As Beta increases, the expected return also increases  IV Rational investors will form portfolios to eliminate systematic risk a) b) c) d)

I & II only II & III only III & IV only I & IV only

Solution: Beta only captures systematic non-diversifiable risk of a security. Therefore, I False II True, III True, IV False. Investors form portfolios to eliminate non-systematic risk

230. Which of the following are among the advantages of money market funds?    

I Provides current income II Provides safety of principal III Notice is required for withdrawal IV Provides capital gains a) b) c) d)

I only I & II only I, II and III only All of the above

Solution: A money market fund invests only in short term money market instruments such as short term debt securities, Treasury bills, banker‟s certificates of deposits and bank acceptances. As the instruments are highly liquid, usually no notice for withdrawal is required. In fact, some money market funds provide quasi checking facilities. Money market funds can pay current income as the instruments mature in the short term. It should also be mentioned that money market funds have low interest rate risk because of the short tenor of the instruments. 231. Which of the following are not characteristics of a closed end fund? I Offers a fixed number of units at the time of the initial public offering II The manager is obligated to repurchase units on request III Typically trades at a discount to its net asset value IV The fund will have to reduce equity allocation in the face of redemptions a) I & III only b) I & IV only c) II & III only Investment Planning – 19Oct 07

43

d) II & IV only Solution: Only I and III are true. II There is no need for the manager to repurchase units. IV Redemptions do not affect a close end fund. The number of units stays the same. 232. Mr A is 30 years of age and intends to retire when he has Rs.400,000. He believes that he can save Rs.10,000 at the end of each year. Being risk adverse, he places his funds in fixed deposits earning only 5.0% pa. His savings at this point is Rs.20,000. At what age will he be able to retire? a) b) c) d)

53 years 51 years 68 years 70 years

Solution: PV is –Rs.20,000, FV is 400,000 with yearly payments of – Rs.10,000. Number years is 21, implying retirement age of 51. 233. What is the net present value (NPV) of a Rs.8.0 million investment with the following cash flows: (Rs. millions) Year 1 Year 2 Year 3 2.0 5.0 6.0 Assume a discount rate of 10% pa for all years. a) b) c) d)

Rs.2.5 million Rs.3.8 million Rs.5.0 million Rs.10.5 million

Solution: Calculations are as follows: (2.0)/(1.1) + (5.0)/(1.1)^2 + (6.0)/(1.1)^3 – 8.0

234. Mr A places his fixed deposit for one year with interest payable at maturity. Mr B places his fixed deposit with another bank with interest payable quarterly. Assuming that the deposit amount is Rs.100,000, and using the same annual rate of 6%, what is the difference in future value after one year? a) b) c) d)

Rs.130.43 Rs.132.54 Rs.134.22 Rs.136.36

Solution: (100,000) x (1 + 0.015)^(4) – (100,000 x 1.06) = 136.36 235. Which of the following assumptions support the use of technical analysis? a) Future performance should be reflective of past performance b) The values of market indices and stock prices are determined based on supply and demand c) Stock prices move in trends that would persist over long periods d) All the above 236. Of the four pairs of portfolios, which pair provides the highest level of diversification? a) b) c) d)

Portfolio 1 & 2: with a correlation coefficient of +0.92 Portfolio 3 & 4: with a correlation coefficient of +0.37 Portfolio 5 & 6: with a correlation coefficient of 0 Portfolio 7 & 8: with a correlation coefficient of -0.78

237. The profit earned by a Mutual Fund will be directly proportional to its The level of the BSE Sensex Assets under Management Investment Planning – 19Oct 07

44

Its operating expenses The sponsor‟s net worth 238. Anil Kumar has two Mortgage Loan options before him. The interest rate, and other conditions are the same for both, except that one has a repayment term of 15 years and the other has a repayment term of 30 years. Anil wants to evaluate the EMIs for both terms. All other conditions being the same, repaying a loan in 15 years instead of 30 would require EMIs that are________. Half the size of the 30-year loan payments. Less than twice as large as the 30 year loan payments. More than twice as large as the 30 year loan payments. Twice as large as the 30 year loan payments. Solution: Suppose a loan of Rs 1,00,000 Interest = 8% EMI for 15 Years=Rs 11682.95 EMI for 30 Years=Rs 8882.74 Rs 11682.95 < 2 * 8882.74 239. You are running a Dividend Yield Fund for a leading Mutual Fund House. The most recent dividend of All Is Fine Business Services common stock was Rs 2.35. The dividends are expected to grow at 4 percent indefinitely. If you are looking at a 12 percent return, how much will you be willing to pay for one share of All Is Fine Business Services? a) b) c) d)

Rs. 24.79. Rs. 29.38. Rs. 30.55. Rs. 32.45.

Solution: M.V of a Stock=[(Dividend*(1+growth rate)] / (Return- growth rate) 240. You are an owner of an apartment complex with 300 units, each of which can fetch Rs. 1000 p.m. as rentals. The apartment complex has an average occupancy rate of 75%. The expenses for maintaining, up keeping the apartment comes to around Rs. 10 Lakh p.a. Based on the concept of capitalized earning approach and assuming that you require a capitalization rate of 10%, how much is the complex worth now? a) Rs. 1.50 crore b) Rs. 1.70 crore c) Rs. 2.00 crore d) None of the above. Solution: Annual rent receivable =300*(1000*12)*0.75 (A) Annual upkeep & maintenance =Rs 10 lakhs (B) Net income annually=Rs 17,00,000 (A-B) Capitalization rate=10% Value of the complex=Net income / Capitalization rate 241. Given the following information about securities A and B: Historical Returns for Securities A B Year 1 10% 18% Year 2 6% 12% Year 3 0% 2% 242. Which of the following are true about Securities A and B? 1) A is more risky because it has a higher standard deviation. 2) B is more risky because it has a higher standard deviation. 3) A has a higher risk-adjusted return. 4) B has a higher risk-adjusted return. Investment Planning – 19Oct 07

45

a) b) c) d)

1 and 3 1 and 4 2 and 3 2 and 4

243. Given the capitalization rate of the following link houses in the BKC area, how much should your client pay for the link house in BKC 4? Comparable Property (1000 sq ft) BKC 1 BKC 2 BKC 3 BKC 4 a) b) c) d)

Annual Net Income (Rs.)

Market Value (Rs.)

14,400 16,800 12,600 15600

342, 800 308,200 210,000 ?

Market Capitalization Rate (%) 4.20% 5.45% 6.00%

Rs. 306,657 Rs. 303,121 Rs. 299,041 Rs. 293,786

Solution: Average market capitalization rate: (4.20% + 5.45% + 6.00%)/3 = 5.22% Market value: Rs. 15,600/0.0522 = Rs.299,041

244. Calculate the yield to maturity of a bond with the following parameters: Face Value : Rs.1000.00 Market Price : Rs.1040.00 Coupon Rate (paid annually) : 7.0% Remaining Term to Maturity : 5 years a) b) c) d)

8.10% 7.69% 7.00% 6.05%

245. Using the Growth Dividend Model, calculate the price of the stock of Company A, with the following information:     

EPS =Rs.10 per share Current Dividend = Rs.8 per share Dividend growth rate = 5.0% Risk free rate = 6.0% Company A risk premium = 7.0% a) b) c) d)

Rs.114 Rs.105 Rs.100 Rs. 62

Solution Using the Growth Dividend Model, the calculations are as follows: R = Risk free rate + risk premium Div1 = Div x (1 +g) Investment Planning – 19Oct 07

46

P = Div1/(R – g) Where P = price Div1 = dividend in year 1 R = required rate of return or discount rate g = constant dividend growth rate R = 0.06 + 0.07 = 0.13 Div1 = 8 x (1 + 0.05) = 8.4 P = 8.4/(0.13 – 0.05) = 105 246. An investor invested Rs.4000 in each of Funds A & B with details as provided below. Both funds subsequently appreciated by 18% based on their NAV.Which of the following statements is/are true?

Fund A Fund B    

Buying 0.96 0.55

Selling 1.02 0.58

NAV 0.95 0.55

I The investor obtained 3,921.57 units in Fund A. II The investor obtained 6,896.55 units in Fund B. III The investor would have achieved a higher return by investing in Fund A alone IV The investor would have achieved a higher return by investing in Fund B Alone a) b) c) d)

I & II only I, II & III only I, II & IV only III only

Solution: As both funds appreciated by the same amount, the investor made identical returns from both funds: Rs.4000 x 18% = Rs.720. There is a misconception that a lower priced fund has a higher return. I is true. Rs.5000/1.02 = 4,901.96 units II is true. Rs.5000/0.58 = 8,620.69 units

247. What is the duration of a zero coupon bond with yield to maturity of 6% maturing in 6 years time? 4.35 5.34 6.00 6.35 248. Calculate the return of an investor who bought units of Fund A on 31/12/06 and sold the units on 31/12/07 with the information provided below? Fund A Price (31/12/06) : Price (31/12/07) : Dividend paid in 2007 : a) b) c) d)

BUY Rs106 Rs.112 Rs. 6

SELL 113 118

NAV 106 112

0.9% 4.4% 11.3% 17.0%

Solution: Total return is calculated as follows: (P2 – P1 + Div)/P1 x 100, (112 – 113 + 6)/113 = 4.4% 249. Bank A quotes the following fixed deposit rates: 1 month 5.3% months 5.6% months 5.8% 12 months 6.2% Investment Planning – 19Oct 07

47

250. Mrs K expects the six-month fixed deposit rate to rise in six months time. She wants to know above what level will the six month rate have to rise in six months time, for her to be worse off, in placing her funds for one year today? a) b) c) d)

6.0% 6.2% 6.4% 6.6%

251. The most appropriate investment for a highly risk averse investor aged 57 is __________ a) An index fund based on the Nifty. b) A diversified equity fund c) A Maharashtra State Government Bond d) A SBI Fixed Deposit 252. In industry life cycle, revenue, margin and profit are expected to peak in the following order: a) b) c) d)

Revenue, margin, profit Margin, profit, revenue Margin, revenue, profit Profit, margin, revenue

253. The broadest measure of inflation is: a) Consumer Price Index b) Wholesale Price Index c) Both a and b

C:\WINDOWS\TEMP\quiz-apr-inv2.html 254. NPV is calculated in the case of a series of ---------------- cash flows. a) b) c) d)

Zero Single Uneven Even

255. The Present Value of a sum of money _________ as the Discounting Rate ______________. b) c) d) e) f)

Remains same, Increases Decreases, Decreases Increases, Increases Increases, Decreases Data Insufficient

256. Classifying an investment as a long term investment depends primarily on; a) the length of time the investor expects to hold the investment. b) the amount of the investment. c) whether a liquid market exists for selling the investment. 257. A period when an economy is experiencing substantial growth and either a declining jobless rate is called_____________. a) b) c) d)

Stagflation Deflation Depression Boom

Investment Planning – 19Oct 07

48

258.In India Futures contracts in ___________ may be settled by delivery. a) Commodities b) Stocks c) Stock Index 259. Money has time value. It derives this value due to existence of several conditions. Which one of the following is not one of the conditions contributing to the existence of this value? a) b) c) d)

The fees and commission sources of the firm Possibility of increase in tax rates over time. Ability to buy/ rent assets generating revenue Cost of foregoing present consumptions

260. You have term deposits of Rs. 4,00,000 with a bank. In order to meet sudden requirements for liquidity and short-term credit, you are applying for an overdraft facility with the bank. What is the rate of interest you will pay on this facility? a) The bank will apply a flat rate of interest on the amount of overdraft allowed to actually utilize. b) The bank will apply a flat rate of interest on the amount of overdraft allowed to you. c) The bank will apply rate of interest linked to the term deposit rate, on the amount of overdraft utilized. d) The bank will apply rate of interest linked to the term deposit rate, on the average amount of overdraft remaining 261. The Nifty has doubled since the last time you advised your client to reduce his equity exposure. The client is annoyed. What might be the most appropriate action to take immediately? a) b) c) d)

Apologize for wrongly forecasting the market Change his asset allocation by increasing his equity exposure Help the client understand the logic of his asset allocation Rebalance his asset allocation by reducing equity investments

262. A professional indemnity policy protects the insured from risk arising out of _____________. a) b) c) d)

Intentional misconduct Misrepresentation of professional competence Negligence Undisclosed conflict of interest

263. Protector International is a financial services firm that specializes in investment advisory services. In its brochure for Financial Planning services, it may state _____________. a) It can offer superior investment returns on customer portfolios and talk of the arrangements to offer advice in other areas b) It has the competence to take care of all Financial advisory requirements of the customer c) Its competence in investment advisory services and the arrangements to offer advice in other areas d) Its Financial Planning services are the best available in the market in light of its investment advisory capabilities and arrangements to offer advice in other areas 264. Which of the following is a concurrent indicator of the phase of the business cycle? a) b) c) d)

Wholesale price Index Index of Industrial production Labor costs and capacity utilization Order levels in the manufacturing sector

265. The effective interest rate earned per rupee _______ as the periods of compounding increase. a) Increases b) Decreases c) Remains same Investment Planning – 19Oct 07

49

d) Decreases for some time and then increases e) Data insufficient 266. What is the main difference between the personal Financial Planning needs of the employed and the self-employed? a) b) c) d)

Attitude to risk/Risk appetite Need to fund children's education Need to fund retirement The extent of any employer-provided pension benefits

267. Immunization protects bondholders from which of the following risk/s: 1) Interest rate risk 2) Reinvestment rate risk a) b) c) d)

3) Maturity risk

1 only 2 only 1 & 2 only 1, 2 & 3

268. The economy is going through a phase of expansion and growth. Industrial production and profitability are high. Your client has a portfolio that is heavily invested in bonds. Which of the following fears of the client is well founded? a) Higher rates of growth will increase demand for funds and interest rates will firm up, leading to fall in bond prices. b) Higher rates of growth will require higher imports and expenses. The government deficits will go up. c) The central bank will try to reduce rates to make funding of business cheaper and reduce costs. That will d) The currency will become convertible and interest rates will rise as a consequence. 269. Ram is a Financial Planner in a large firm. His wife has some large investments in the shares of a few companies. Ram is required to offer views on almost all of these holdings to clients. Under the Code of Ethics and Rules of Professional Conduct _______ a) Ram must disclose the fact to his client(s) so as to make them aware of any potential conflict of interest b) Ram has to disclose these holdings only to his employers, if required by the firm's internal compliance rules c) Ram need not follow any code of ethics and rules of professional conduct. d) Ram will not violate the Code and the Rules if he does not disclose his wife's holdings 270. How are financing costs included in NPV and IRR calculations? a) b) c) d)

By including them in the interest payments. By considering the interest rate in the setting of the discount rate As a tax deduction By including them in the earnings

271. According to fundamental analysis, which phrase best defines the intrinsic value of a share of common stock? a) b) c) d)

The par value of the common stock. The book value of the common stock. The liquidating value of the firm on a per share basis. The discounted value of all future dividends.

272. The term .Efficient Frontier. is contained in________. a) Technical Analysis b) Modern Portfolio Theory c) Value Investing Theory Investment Planning – 19Oct 07

50

273. A major difference between load and no-load funds is a) Marketability: no-load funds can be traded more readily. b) Acquisition cost: load funds cost more than their NAVs. c) Performance: load funds do better. 274. The Reliance fund trades on the NSE. Its recent price is Rs.10, but its NAV is Rs.12. We know then a) The fund is closed-end, selling at a discount. b) The fund is open-end, selling at a premium. c) The fund is closed-end, selling at a premium. 275. If your investment goal is simply to match the market, should buy a(n) a) Growth Fund b) Money market fund. c) Index fund. 276. You are evaluating a fund. What activity would you typically not undertake in this effort? a) Calculate or find the fund's rate of return. b) Calculate the fund's NAV. c) Find the fund‟s turnover ratio and administrative expenses-to-assets ratio. 277. The Cholamandalam Fund‟s rate of return was 9%, while the market return was 15%. Cholamandalam‟s beta was 0.5. a) Cholamandalam's management out-performed the market on a risk-adjusted basis. b) Cholamandalam's RAROR was 11.5%. c) Cholamandalam's management under-performed the market on a risk-adjusted basis. 278. You are considering taking a passenger with you when you go home over Christmas break. She lives 100 kms out of your way, and the total trip is 500 kms. She has offered Rs.50o for the service. You estimate the total cost of the trip at Rs.300o. You should a) Reject the offer since 20% (1000/5000) of Rs.3000 is greater than Rs.500. b) Accept the offer if the opportunity costs of the trip is greater than Rs.2500. c) Accept the offers if marginal costs associated with 100 kms are less than Rs.500. 279. liquidity ratio of 2.0 tells us that the family has a) Rs.2 in liquid assets for each Rs.1 in total liabilities. b) Rs.2 in liquid assets for each Rs.1 in current liabilities. c) Rs.2 in liquid assets for each Rs.1 of total expenses. 280. In India, Preference shares may be issued for a maximum number of ___________ years. a) b) c) d)

12 15 10 20

281. Determination of Residential status for the tax purpose is applicable on: i. Individual a) b) c) d)

ii. Firm

iii.Company

iv. HUF

i,ii, i,ii, iii i only i, ii, iii, iv

282. A growth-oriented non-divided paying share is bought for Rs.250 and sold for Rs. 450 after 51 Investment Planning – 19Oct 07

5 years, the compound annual growth rate is: a) b) c) d)

14.86% 12.47 % 11.50% 10.71%

283. The call option strike price on a share is Rs. 500 and the current share price is Rs. 550. The call option premium is Rs. 60. The time value of the option is: a) b) c) d)

60 10 30 15

284. Mr. A deposits Rs. 10,000 in his own PPF account and same amount in his wife.s account. How much maximum amount can he deposit in his nephew.s name? a) b) c) d)

Rs. 20,000 Nil Rs. 70,000 Rs. 60,000

285. A Rs.100 par value bond having 10 % coupon rate will mature after 7 years. Find the value of the bond if the discount rate is 8 %. a) b) c) d)

109.85 111.41 108.75 110.60

286. Consider a portfolio of two investments viz. A & B. The sum total of volatility of A and B respectively, represented by standard deviation of the two investments, will be equal to the volatility of the portfolio as a whole if _________________. a) b) c) d)

A and B have a correlation of Zero A and B have a correlation of 1 The portfolio is equally divided between A and B The return on the portfolio is equal to the sum of returns of A and B

287. Which of the following is a correct interpretation of the Rules of Conduct pertaining to the Ethic of Confidentiality? a) A Member must when requested by the client, provide to a person authorized by the client, all original documents prepared or received by the Member in undertaking the advisory task b) A Member owes to the Member's partners or co-owners a responsibility to act in good faith (expectations of confidentiality) only while in business together, not thereafter c) The Member shall maintain the same standards of confidentiality to employers as to clients d) Under no circumstance, will any Member divulge any information or knowledge regarding the FPSB India or its members that they may know or be exposed to 288. Mr. Rajan's investment portfolio comprises Rs.2 lakh in equity, Rs.5 lakh in debt and Rs.1 lakh in his bank current account. Over one year the returns on equity and debt are 5% and 12%. At the end of the year to maintain his current asset allocation, he needs to _____________. a) b) c) d)

Do nothing. He needs to move Rs, 10000/- from equity and Rs. 60000/- from debt to cash. He needs move Rs.7500/- to equity from debt and Rs. 8750/-to cash from debt He needs to invest Rs. 70000/- in debt and equity.

289. A 10 year 8.0% bond (Face Value- Rs.1000, interest payable semi-annually) maturing 6 years 52 Investment Planning – 19Oct 07

from today is available at a yield to maturity of 6.0%. It is likely to be priced at _______________. a) b) c) d)

Rs. 1100 Rs. 1149 Rs. 1168 Rs. 1498

290. In India, Mutual Funds have recently moved to the concept of ___________AUM calculation. a) Monthly average b) Month end c) Fortnightly average 291. Raykar is an accomplished Financial Planner and is also an expert on derivatives and high yielding bonds. He understands client requirements well and is able to come up with appropriate portfolio restructuring ideas for clients. He believes in quickly moving clients from one investment to another through a dynamic process of research and recommendations. What according to the Rules relating to the Code of Ethics is the most applicable in this case? a) He does not violate the Rules if he explains to the client the reasons and is able to show that the moves are appropriate to the client b) He does not violate the Rules since he conducts and has access to research and advises on products relevant to clients based on an understanding of their requirements c) He does not violate the Rules since he is an acknowledged expert and knows what is best for his clients d) He violates the Rules as it amounts to active churning of client portfolios

292. Mrs. & Mr. Arora are aged 55 and 58 years respectively. Both expect to work till they turn 65. Their only goal is to fund their retirement. Which of the following is likely to be an appropriate asset allocation strategy for them? a) 10% sectoral equity, 20% diversified equity, 30% long-term debt, and 40% medium term debt b) 20% Sectoral equity, 60% diversified equity, 20% long-term debt c) 30% Sectoral equity, 30% diversified equity, 40% cash/ liquid investments. d) 80% long-term debt, 20% medium term debt 293. In analyzing the position of a portfolio in terms of risk/return on the capital market line (CML), superior performance exists if the fund's position is the CML, inferior performance exists if the fund's position is the CML, and equilibrium position exists if it is the CML. a) b) c) d)

Above; on; below. Above; below; on. Below; on; above. Below; above; on.

294. Which of the following best describes the investment characteristics of a high-quality long-term municipal bond? a) b) c) d)

High inflation risk; low default risk. Low inflation risk; high market risk. Low inflation risk; low default risk. High inflation risk; high market risk.

295. An Asset Management Company must have a minimum corpus of Rs.______________ crores. a) 5 Investment Planning – 19Oct 07

53

b) 15 c) 25 d) 10 296. A 5 year annual annuity has a yield of 6%. What is the duration? a) b) c) d)

2.88 years 2.55 years 3.16 years 1.35 years

297.The price of Stellar Ltd. is currently Rs.40. The dividend next year is expected to be Rs.4.00. Required return on the stock is 12%. Find the expected growth rate under the Constant Growth model. a) b) c) d)

2.00 % 2.25 % 1.90 % 2.75 %

298. Data on two stocks is given for 2 different scenarios below. Find Beta of both stocks Market Return Infocomm Ltd. FMCG Ltd. 5% 3% 7% 15% 25% 12% a) b) c) d)

1.8, 0.60 2.5, 0.75 1.5, 0.25 2.2, 0.5

Solution: Beta of Infocomm is (25-3)/ (15-5) =2.2; Beta of FMCG = (12-7)/ (15-5) = 0.5

299. Data on a mutual fund is given: Fund Name Mean Return A 10% Market Index 16%

Std. Deviation 25% 20%

Beta 0.75 1.00

The risk free rate is 9 %; Calculate Treynor, Sharpe and Jensen measures. a) b) c) d)

1.05, 0.10, 4.25 1.33, 0.04, -4.25 1.10, 0.15, -3.75 1.46, 0.09, 3.75

Solution: Working Note: Treynor Ratio = (Rp-Rf)/ Beta; Hence 1.33; Sharpe Ratio = (Rp . Rf)/Standard Deviation; Hence 0.04; Jensen = Rp-[Rf + B(RM .RF)] ; It measures the risk adjusted portfolio return. It is also known as Jensen.s Alpha; Hence -4.25 300. ABC Ltd. is willing to prepay your Cumulative Fixed Deposit with them, without any penalty and with all the accumulated interest (compounded half yearly). You had invested Rs. 4000 with them 3.5 years back. If they are giving you back Rs. 4985, what is the annualized rate of interest you have earned? a) b) c) d)

6.40% 3.2%. 6.5%. 7.2%.

Solution: (((((4985/4000)^(1/7))-1)*100)*2) 301.. Which of the following is a tort of negligence? Investment Planning – 19Oct 07

54

a) Mr. Joy was playing golf. He swings a new golf club on the fairway and the head of the club flies off, and hit another golfer who was standing 20 feet away. b) Mr. Vishal takes medication that he knows makes him drowsy and then proceeds to drive. He gets into an accident injuring the passengers in another car. c) Mrs. Jaya locks Ms. Rani in a room to prevent him from leaving the building d) Mrs. Priti experienced a sudden surge of chest pain while driving, which causes her to lose control of her car and hit another car. 302.. Any possible occurrence which may have a negative financial implication, can be plotted on a graph with X axis measuring the frequency (low-high) and Y axis measuring the financial impact (lowhigh). You can view the classification in four quadrants. Quadrant Quadrant Quadrant Quadrant

I - Low frequency, Low Impact II - Low frequency, High Impact III - High frequency, High Impact IV - High frequency, Low Impact

It would not be practical to purchase insurance for events falling in _________________. a) b) c) d)

Quadrant I & IV Quadrant I, II & IV Quadrant I, III & IV Quadrant III

303. _____________funds pay a Dividend Distribution tax on dividends. a) Equity b) Index c) Debt

304. The current annual dividend of ABC Corporation is Rs.2.00 per share. Five years ago the dividend was Rs.1.36 per share. The firm expects dividends to grow in the future at the same compound annual rate as they grew during the past five years. The required rate of return on the firm's common stock is 12%. The expected return on the market portfolio is 14%. What is the value of a share of common stock of ABC Corporation using the constant dividend growth model? (Round to the nearest Rupee.) a) b) c) d)

Rs.11. Rs.17. Rs.25. Rs.54

Solution: PV = Rs.1.36, FV = (Rs.2.00), N= 5, i = 8.02 Value of common stock = d1/ RRR – g = 1.082 (2.00)/ 0.12 – 0.08 = 2.16/0.04 = Rs.54.00 RRR = Required Rate of Return, g = growth rate 305. Bond A has a 6% annual coupon and is due in 2 years. Its value in today's market is Rs. 900. Bond B has a 10% annual coupon and is due in 4 years. It is priced to yield 12%. Bond C is a 9% zero-coupon bond priced to yield 11% in 8 years. The yield to maturity of Bond A is closest to: a) b) c) d)

9.90%. 10.40%. 10.90%. 11.90%.

Solution: Yield to maturity for bond A: N = 2, PV = (Rs.900), PMTOA = Rs.60, FV = Rs.1,000, i = 11.90998 Investment Planning – 19Oct 07

55

306. Bond A has a 6% annual coupon and is due in 2 years. Its value in today's market is Rs.900. Bond B has a 10% annual coupon and is due in 4 years. It is priced to yield 12%. Bond C is a 9% zero-coupon bond priced to yield 11% in 8 years. Assuming that the duration of Bond A is 1.94 years, which of the following statements about the effect of a 1% decline in interest rates is true? a) b) c) d)

Bond C, having a longer duration than Bond A, would have a larger percent increase The percent change in price of a bond is independent of the duration of a bond. It is not possible to determine the percent change in price of Bond A versus Bond C Bond A would have a greater percent change in price than Bond C because it has a shorter duration.

307. A perfectly diversified portfolio will fully eliminate ______________ risk. a) Systematic b) Unsystematic 308. __________________measures the caliber of the fund manager. a) Beta b) Delta c) Alpha

C:\WINDOWS\TEMP\quiz-apr-inv3.html 309. The new Senior Citizen.s Savings Bond Scheme offers ________ % Interest. 8.5 9 9.25 8.75 310. The maximum amount that can be invested in Public Provident Fund is Rs._________. a) b) c) d)

70000 60000 80000 90000

311. _____________ is regulated by the Reserve bank of India. A. Bank Deposit Rates; B. Bank Lending Rates; C. Certificate of Deposit Rates a) b) c) d)

A B C None

312. The recent Union Budget exempts ______ from Long Term Capital Gains tax subject to certain conditions. a) b) c) d)

Equity Shares Debt Mutual Funds Property Gold

313. The deduction from Gross Total Income available u/s 80E is Rs.___________. a) 40000 b) 45000 c) 50000 Investment Planning – 19Oct 07

56

314. _________ is / are governed by SEBI. a) b) c) d)

Mutual Funds Stock Brokers Portfolio Managers All of the above

315. A person can be qualified as an Associate Financial Planner after he/she passes modules of the CFP Certification Course. a) b) c) d)

6 2 3 5

316. The first step of the financial planning process is ___________. a) b) c) d)

Evaluating the various Alternatives Data gathering and goal setting Establishing the Client Planner relationship Plan Review

317. How many years will it take for a sum of Rs.10000 to double if the rate of return is 9% p.a.? a) b) c) d) e)

9.5 8.5 10 9 8

318. If the post tax rate of return on an investment is 8% and the inflation rate is 5% the real rate of return is___ a) b) c) d) e)

3.5% 3.0% 2.86% -3.0% 2.74

319. The key reference rate for inter-bank overnight borrowings in Mumbai is known as a) MIBOR b) MIBID c) LIBOR d) LIBID 320. This relationship between the trustees & the unit holders of a mutual fund shareholders is called a __________ relationship a) b) c) d)

Contractual Fiduciary Moral obligatory None of the above

321. Selling the wrong type of policies in order to earn higher commissions will render an insurance agent to be held liable ______________ a) Under law of tort b) for professional misconduct c) for professional negligence d) under the SEBI Act Investment Planning – 19Oct 07

57

322. In portfolio management, a constant ratio plan is one that a) Restores asset holdings at the end of a period to target weights. b) Buys different kinds of securities in a fixed proportion. c) Pays out dividends in a constant ratio to the portfolio‟s asset values. 323. As interest rates rise bond prices _____ and the longer their maturities the _______ the price change. a) Rise; lesser b) Fall; lesser c) Fall; greater 324. Which item below is not a method recommended for acquiring securities? a) Dividend reinvestment plans b) Rupee cost averaging c) Averaging alpha and beta weights 325. KM, Inc. has a beta of 1.5. The risk-free rate is 5%, and the market risk premium is 8%. Thus, a) KM has a required return of 6.5% and an expected return of 8%. b) KM has a required return of 15.5%. c) KM has a required return of 17.0%. 326. NASDAQ is a) A section of the NYSE where technology stocks are traded. b) The trading symbol for a racetrack company listed on the NYSE. c) An electronic trading system used by securities firms.

327. An equity mutual fund has an NAV of Rs. 15 on January 1, 2006. You invest Rs. 15000 on that day. You sell these units at an NAV of Rs. 25 on January 15, 2007. The entry load is 2.25% while the exit load is 0.50%. You are in the 30% tax bracket. Your post-tax return on the same is _________ % a. b. c. d.

66.66 62.18 46.66 42.18

328. The thumb rule states that we should choose an investment with a _______ IRR. a. b. c. d.

Lower Higher Equal Data insufficient

329. Seema and Arun are co-applicants of a mortgaged house. They are on the verge of a divorce. The Housing Finance Company will ___________. a) b) c) d) e)

not interfere as long as the EMIs are being paid on time repossess the house after divorce insist on the house being transferred to one of them mediate reconciliation between the couple. Increase the interest rate in order to compensate for the increased risk

330. Domestic GOI bond holders (holding them up to Maturity) have to deal with ___________ risk. a) Volatility b) Default c) Inflation Investment Planning – 19Oct 07

58

d) Price e) Currency 331. Refinancing is ______________. a) b) c) d) e)

Borrowing at lower cost in order to pay off higher cost debt Repaying debt by selling off assets Lending at a higher rate of interest Securitizing your receivables None of the above

332. ________________ Asset Allocation is not a text book Asset Allocation Model. a) b) c) d) e)

Tactical Discretionary Strategic All of the above None of the above

333. Jack and Jill approach you to be their Financial Planner their funds are limited and their needs are many. Some of their needs are: a) To start an investment plan for funding their child.s education; b) To set up a Testamentary Trust for their child; c) To set up a contingency fund amounting to 3 months of living expenses d) To start saving for retirement; e) To purchase life and health insurance. Arrange these needs in the descending order of priority. a) b) c) d)

ceadb debca bdeac beacd

334 For a nominal interest rate of 6% payable monthly, quarterly, and semi-annually, the effective rates respectively ould be _______________. a) b) c) d)

6.04, 6.02, 6.01 6.16, 6.13, 6.09 6.10, 6.07, 6.03 6.11, 6.08, 6.06

335. A 10 year 9 % Bond (Face Value of Rs.100, interest payable annually) maturing 3 years from today is available at a YTM of 5.8%. Therefore the current price is ___________. a) b) c) d)

152.50 154.10 153.32 152.00

336. A one-time principal repayment for a bond is known as a ____________ repayment. a) b) c) d)

Laddered Step-Up Step-down Bullet

337. Arrange the following financial planning functions into the logical order in which a professional financial planner performs these functions. 1. Interview clients, identify preliminary goals. 2. Monitor financial plans. 3. Prepare financial plan. Investment Planning – 19Oct 07

59

4. Implement financial strategies, plans, and products. 5. Collect, analyze, and evaluate client data. a) b) c) d) e)

1, 3, 5, 4, 2. 5, 1, 3, 2, 4. 1, 5, 4, 3, 2. 1, 5, 3, 4, 2. 1, 4, 5, 3, 2.

338 A model that describes the relationship between risk and expected return and that is used in the pricing of risky securities is better known as _________ a) b) c) d)

Efficient Market hypothesis CAPM Security Market Line Beta Model

Four Marks Questions For question no. 339 and 340.The stock of Maxi Limited performs relatively well to other stocks during recessionary periods. The stock of Taxi Limited, on the other hand, does well during growth periods. Both the stocks are currently selling for Rs.100 per share. You assess the rupee return (dividend plus price) of these stocks for the next year as follows:

Probability Return on Maxi‟s stock Return on Taxi‟s stock

Economic Condition High Growth Low Growth 0.3 0.4 100 110 150

130

Stagnation 0.2 120

Recession 0.1 140

90

60

Calculate the standard deviation of investing: 339. Rs.1, 000 in the equity stock of Maxi Limited a) b) c) d)

120.5 124.6 116.6 123.4

340. Rs.1, 000 in the equity stock of Taxi Limited a) b) c) d)

301.4 322.6 291.4 296.8

Investment Planning – 19Oct 07

60

341. Rs.500 each in the equity stock of Maxi Limited and Taxi Limited. a) b) c) d)

90.6 104.6 78.4 89.6

342. You purchase One Contract of the September Call option for Dr. Reddy‟s Labs at Rs 1400 strike price for a premium of Rs. 100 . The price on the date of contract expiry is Rs. 1600. The market lot is 200 shares. Your profit on the above transaction is _______ %. Ignore transaction costs. a) b) c) d)

50 25 15 10

343. Sanjeev invests Rs.5000 in a Bank Deposit today @ 8% p.a compounded monthly.. He hopes that this investment will enable him to fund his college education (estimated to cost Rs.9000) which commences after 4years. What will be the value of this investment in four years? a) b) c) d)

6802 6870 6878 6925

344. Sudha invests Rs.5000 per year (at the beginning of each year) for 5 years @ 5% p.a. in a bank deposit. She then withdraws the accumulated sum over a period of 3 equal annual installments. What is the value of the deposit at the end of 5 years and the quantum of withdrawal each year? a) b) c) d)

28505, 9954 29010, 10656 29568, 11054 28804, 10042

345. Amar wants to purchase a Car 5 years from now. His investments are currently worth Rs.50,000/- and he intends to contribute Rs.10,000/- at the beginning of every six months period to fund his purchase. Assuming that the annual investment rate of return is 8% compounded semi annually, what will be the value of the investment in five years time? a) 1,98,875 b) 1,95,555 c) 1,97,240 346. Neeta wants to accumulate Rs.1, 50,000 in three years time for a one month trip to the USA. Assuming she can get an 8% annual return on her investments, compounded quarterly, how much must she invest today in order to achieve her goal? a) 117591 b) 119487 c) 118274 347. John has estimated that the following will be his outgoings over the next few years: End of Year Cash Outflow a) b) c) d) e)

1st Rs.10000 2nd Rs.15000 3rd Rs.12000 4th Rs.13500 5th Rs.11000

348. If John wants to cater to these cash outflows, how much should he have today, assuming an annual rate of return of 5%? Investment Planning – 19Oct 07

61

a) b) c) d)

54126 53220 52483 50483

349. Mr. John has purchased 100 convertible debentures of Essar Oil on 1/1/94 at Rs.500 each. 40% of the value of the debentures is convertible into one share of Rs.50 each after seven years. Mr. John exercised his option on 1/4/2001 and received 100 shares. Compute the cost of acquisition of these shares. a) b) c) d)

200 205 195 185

350. Revenue over the past four years are given below: st 1 year 100 nd 2 year 120 rd 3 year 150 th 4 year 200 Calculate the CAGR: a) b) c) d)

20% 18.9% 25% 25.9%

Source: Quiz- Aug – Inv 3 351. The maxim “buy term and invest the balance” may not be a feasible proposition for many investors for the following reasons EXCEPT for: A. it may be difficult to achieve sufficient diversification in the invested assets B. it may be difficult to achieve a suitable investment portfolio with the desired risk reward relationship C. it may be possible to consistently outperform the investment returns earned by an established life office D. that the insurer offers capital guarantees on cash values 352. An individual may prefer investment-linked assurance to conventional assurance policies for the following reasons EXCEPT A. he has some direction over the investment of his premiums B. he is informed of the expenses charged for the services provided C. he is attracted by the guaranteed surrender values offered under these contracts D. he prefers the switching facilities available under such contracts 353. Which of the following does not constitute a valid charge on the premiums paid for an investment-linked life assurance policy? A. bid-offer spread B. front-end charges C. recurrent fund related charges D. surrender penalty 354. You are an independent financial planner and are faced with the problem of suggesting a suitable insurer for a client who is in very good health. Assuming that a temporary assurance policy is needed for this client, you would recommend an insurer Investment Planning – 19Oct 07

62

A. with a stringent underwriting policy B. with a lax underwriting policy C. with very few causes of death excluded D. with extensive causes of death excluded 355. Identify the class of assurance for which an insurer, in a costing exercise, must provide for future anticipated improvements in mortality A. investment-linked endowment assurance B. conventional endowment assurance C. group life assurance D. life annuities 356. A type of risk with high frequency but low severity is probably best handled by: a) Self-insurance b) Insurance c) Avoidance d) Transfer 357. 'Consideration' under the law is a return promise to: (1) Do certain things (2) Abstain from doing certain things (3) Forbear some acts (4) Accept an offer a) (1), (2), and (3) only b) (1), (2), and (4) only c) (1), (3), and (4) only d) (2), (3), and (4) only

358. Frank has an indemnity policy in respect of his house. The house was valued when he first took out the policy three years ago at Rs.80,000 and insured for that amount. The policy has been renewed each year since, without alteration. If the house is destroyed by fire and the cost of rebuilding it would be Rs.110,000, how much is Frank likely to recover? a) Rs.58,182 b) Rs.80,000 c) Rs.88,000 d) Rs.110,000 359. A client explains that she only wants a life insurance policy that will cover her family against financial risk over the next five years, while she still has dependent children and a large mortgage. It is unlikely her income will increase over this period. What type of insurance is she looking for? a) Pure endowment assurance b) Term insurance with a level premium c) Single premium life insurance d) Whole of life assurance to be made paid up after five years 360. The 'principle of subrogation' means: a) An insurer may bring an action against a third party in the name of the insured after satisfying a claim b) An insured may not take legal action in respect of a matter which is the subject of a claim c) An insurer undertakes to take legal action on behalf of its insured and to pay to the insured the net proceeds of such legal action d) An insurer which has satisfied a claim may recover from the insurer of a third party who admits liability Investment Planning – 19Oct 07

63

361. Which of the following is true regarding the ownership of life insurance? a) A policy can only be issued to the insured b) Generally, assigning a policy requires proof that the insured is still „insurable‟ c) Only a person with an insurable interest can be named as a beneficiary d) The owner can assign the policy to whomever he or she chooses, even if the assignee has no insurable interest 362. Prakash has a premium due on a motor insurance policy. He calls the agent and requests for an extension. The agent assures him that the insurer usually gives a grace period of one week for such payment. Prakash has an accident. The insurer cannot deny liability for the loss on the grounds that the premium was not paid because of the doctrine of: a) Waiver b) Estoppel c) Agency d) Utmost good faith 363. A reversionary bonus is: a) Any profit distributed to holders of life policies who cash in those policies b) Generally profit that will be distributed on the same terms as the sum assured c) A distribution of income to shareholders d) Profit distributed to holders of unit-linked policies 364. Professional indemnity insurance protects a financial advisor who has been negligent in giving investment advice: a) Only if the advisor is liable under statute b) Only if a contract exists between the advisor and the investor c) Only if the investor relies on the advice d) Only if the advisor has not included a disclaimer of liability in the contract with the investor 365. Deepika purchased a dining room set for Rs.2,800 and insured it on an actual cash value (ACV) basis. At the time the dining room set was destroyed by a covered peril, the set was 40 percent depreciated. A replacement set will cost Rs.3,000. Assuming no deductible, how much will Deepika collect from her insurer? a) b) c) d) 366.

Rs.1,800 Rs.3,000 Rs.1,880 Rs.2,800

Property insurance contracts have all of the following distinct legal characteristics EXCEPT a) b) c) d)

They are personal contracts. They are contracts of adhesion. They are bilateral contracts. They are aleatory contracts.

367. In which section of an insurance contract will you find provisions that qualify or place limitations on the insurer‟s promise to perform? a) b) c) d)

Exclusions Definitions Conditions Insuring agreement

368. Which of the following statements is (are) true about the insuring agreement in an insurance policy? i. The insuring agreement provides a description of the property or activity to be insured. ii. The insuring agreement can be written on an “all risks” basis or on a “named-perils” basis. Investment Planning – 19Oct 07

64

a) b) c) d)

Both I and II Neither I nor II II only I only

369. Saif and his wife Amrita are legally separated. The couple owns a vacation cabin. Saif purchased a Rs.25,000 property insurance policy on the cabin. Unaware that Saif had purchased this coverage, Amrita purchased a Rs.50,000 property insurance policy on the cabin. While both policies were in force, a Rs.12,000 covered loss occurred. The insurers agreed to settle the claim on a pro rata basis. What is each insurer‟s liability? a) b) c) d)

Saif‟s insurer pays Rs.3,000 and Amrita‟s insurer pays Rs.9,000. Saif’s insurer pays Rs.4,000 and Amrita’s insurer pays Rs.8,000. Saif‟s insurer pays nothing and Amrita‟s insurer pays Rs.12,000. Saif‟s insurer pays Rs.12,000 and Amrita‟s insurer pays nothing.

370. Under the needs approach of determining the amount of life insurance to purchase, one consideration is providing income to the surviving spouse and children during the one- or two-year period following the breadwinner‟s death. This period is called the a) Accumulation period. b) Dependency period. c) Readjustment period. d) Blackout period. 371. Sheila would like to purchase cash value life insurance policy. She is concerned, however, that if she becomes disabled she will be unable to pay the premiums as they come due. What provision can Sheila add to her policy to address this concern? a) b) c) d) 372.

Reinstatement provision Waiver-of-premium rider Accelerated death benefits rider Guaranteed purchase option

Which of the following statements is (are) true with respect to life insurance policy loans? i. Interest must be paid on life insurance policy loans. ii. If a policy loan has not been repaid when the insured dies, the outstanding loan balance I s deducted from the proceeds paid to the beneficiary. a) b) c) d)

II only I only Neither I nor II Both I and II

373. Which of the following statements is (are) true with respect to the traditional net cost method of determining the cost per thousand of cash value life insurance? i. The traditional net cost per thousand per year can be a negative number. ii. The traditional net cost method ignores interest that could have been earned on the premiums by investing them elsewhere. a) b) c) d)

I only II only Both I and II Neither I nor II

374. Bob and Jasmine is a married couple who are both 67 years old. Bob and Jasmine purchased an annuity covering both of their lives. The settlement option will provide payments until Bob and Jasmine are both deceased. The settlement option Bob and Jasmine selected is a (n) a) Installment refund option. b) Life income with guaranteed payments option. c) Cash option. d) Joint-and-survivor annuity option Investment Planning – 19Oct 07

65

375. Shelly purchased a major medical policy. The policy has a calendar-year deductible of Rs.500 and 80-20 coinsurance. Shelly was hospitalized with a covered illness on January 23rd. This hospitalization was his first claim under the major medical policy for the calendar year. His covered medical expenses were Rs.20,500. How much of this amount will the insurer pay, and how much will Shelly be required to pay? a) b) c) d)

The insurer will pay Rs.16,900 and Shelly will pay Rs.3,600. The insurer will pay Rs.16,000 and Shelly will pay Rs.4,500. The insurer will pay Rs.15,900 and Shelly will pay Rs.4,600. The insurer will pay Rs.20,000 and Shelly will pay Rs.500.

376. What lump sum contribution made now into a deferred annuity scheme will provide to 20-year old males annual payments commencing at age 55 of Rs. 1,200 and escalating thereafter at an annual rate of 2% per annum for the next 20 years, if the funds under such a scheme earn 6% per annum rate of interest throughout? A. Rs.2,221 B. Rs.1,898 C. Rs.2,137 D. Rs.2,002 377. Moral hazard is likely in insuring each of the following risks EXCEPT: A. A company that had made many claims under its fire insurance policy B. An individual purchasing life annuity at his retirement. C. An individual purchasing life insurance on his own life D. An individual purchasing life insurance on his wife‟s life

378. An insurance contract is voidable if it can be established that the policyholder A. did not state his correct date of birth B. did not provide precise information about his parents C. died within the contestable period D. did not furnish accurate information about his current state of health 379. The following are important factors in the determination of premiums for disability income policies EXCEPT: A. The elimination period B. The definition of disability C. Preexisting conditions D. The insured‟s occupation 380. What lump sum contribution made now into a deferred annuity scheme will provide to 20-year old males annual payments of Rs.1,200 commencing at age 55 for the next 20 years, if the funds under such a scheme earn 6% per annum rate of interest throughout? A. Rs.1790 B. Rs.1898 C. Rs.2006 D. Rs.3122 381. For an initial lump sum consideration of Rs.5,295 paid at age 20, a deferred life annuity scheme offers the following benefits: On attaining age 55, an annuity certain for 10 years and a life annuity thereafter, of Rs. 1,200 per annum. Assuming the annuitant dies at his 75th birthday, calculate the internal rate of return implicit in the promises made. Investment Planning – 19Oct 07

66

A. 3.1 % B. 3.3 % C. 3.5 % D. 3.7 % 382. One of the key criteria for the approval of applications made IRDA for the conduct of insurance business in India, is that the applicant must have a large capital base. This large capital base is necessary for the following reasons EXCEPT: A. To absorb the losses that may result from charging inadequate premiums. B. To absorb the catastrophic losses that may arise in some years. C. Insurance business is capital intensive. D. A large capital base is a pre-requisite for achieving economies of scale. 383. The following is an abridged version of the balance sheet of a large life insurance company: Balance Sheet of ABC as at 31Dec, 2006 (Rs.‟000,000) Liabilities Shareholders‟ Funds 800 Statutory Liabilities 3,400 Current Liabilities 50 Total 4,250 Backed By Indian Government Securities 700 Corporate Bonds 200 Ordinary Shares 2,050 Properties 800 Policy Loans 500 Total 4,250

A measure of the company‟s financial strength is: A. Rs. 4,250,000,000 B. Rs. 3,450,000,000 C. Rs.3,400,000,000 D. Rs. 800,000,000 384. The proceeds of Rs.150,000 from a life insurance policy is to be disbursed under a settlement option using an annuity due with term certain of 15 years. Assuming that a 4% per annum interest is appropriate, the level annual payments arising under this arrangement is: A. Rs.13,491 B. Rs.12,972 C. Rs.7,203 D. Rs.7,491 385.

The waiting period under a disability benefit policy refers to a. The period of time that must elapse before the policy benefit will commence b. The period of time that must elapse before settlement of the claim after submission. c. The period of time that must elapse before the issue of the policy document. d. The period of time that must elapse before the proposal can be considered.

386. Seema and Arun are co-applicants of a mortgaged house. They are on the verge of a divorce. The Housing Finance Company will ___________. a) not interfere as long as the EMIs are being paid on time b) repossess the house after divorce c) insist on the house being transferred to one of them d) mediate reconciliation between the couple. Investment Planning – 19Oct 07

67

e) iIncrease the interest rate in order to compensate for the increased risk 387. In the event of a claim, an indemnity contract pays to the insured an amount equal to A. the sum insured B. the extent of loss subject to a maximum equal to the sum insured C. the extent of loss without any limits D. a previously agreed amount 388. For a disability income policy, which of the following factors are important in its premium determination? I. Replacement ratio

II Definition of disability

A I only B II only C I and II D Neither I nor II 389.

In controlled Third party captive business, retailers are underwriting a. Credit life, health and disability; extended warranty b. Subcontractor coverages c. Joint ventures d. Credit life, health and disability; mortgage insurance, title insurance , auto and property.

Use the information. below to provide answers to Questions 390 to 392 A client of yours, who is now aged 35 years, states that his retirement objective is to provide for himself at retirement at age 55, annual incomes of 2/3 of his last drawn salary. He further states that he expects this income for 20 years after retirement. His current annual salary is Rs. 60,000 and this escalates at 6% per annum throughout his career, with the annual escalations occurring on his birthdays. Assuming an 8% per annum interest rate throughout,

390. Calculate his last drawn annual salary A. Rs.128,400 B. Rs.181,536 C. Rs.192,428 D. Rs.132,000 391. Calculate the present value of the retirement income A. RS.225,800 B. RS.250,900 C. RS.275,327 D. RS.305,780 392. Calculate the level annual amount that he must deposit until age 54, assuming that the first is due now, to fund for the stated retirement income. A. RS.22,800 B. RS.25,965 C. RS.28,890 D. RS.31,234 393. Participating ordinary life policies with a sum assured of Rs. 10,000 are issued by two life assurance companies, Office 1 and Office 2, to lives aged 35 with the following cost data: Data Annual premiums Accumulated value of Investment Planning – 19Oct 07

Office 1 Rs. 230 1613

Office 2 Rs. 290 1700 68

Total bonuses for 20 years at 6% pa Cash value at end of 20 years

3620

4000

394. Based on the above information and assuming a 6% per annum interest rate, the annual surrender cost index for each Rs.1,000 sum assured at the end of 20 years A. For Office 1 is greater than Office 2 B. For Office 2 is greater than Office 1 C. For Office 1 is equal to Office 2 D. Is not determinable

395.

Calculate surrender value:

Date of commencement – 5.10.1990 Date of last premium – 5.4.2000 Date of birth – 1.5.1950 Mode of installment premium – half yearly Plan – Endowment with profit – 50 years Sum assured – Rs.1, 20,000 Bonus accrued from 3/91 to 3/2000 is Rs.700 per 1, 000 sum assured Accidental benefit – Rs.2 per thousand sum assured S.V. factor – 25% a. b. c. d.

6000 27,000 6,240 27,240

Solution: Paid-up value- No. of premiums paid X sum assured /No. of premium payable + bonus if any. Surrender value Paid-up value X S.V. factor/100 Paid-up value: 20 X 1, 20,000/100 = Rs.24, 000 + 84, 000 (bonus for 10 years Rs.700 per 1, 000 S.A) = Rs.1, 08,000 Surrender Value: 1,08,000 X 25/100 = Rs.27, 000 Therefore surrender value is Rs.27, 000. 396.

Calculate the premium:

Tabular premium: Rs.33.10 per sum assured. : Rs.2 less for yearly mode. : Re.1 less for half yearly mode. : Rs.3 less for sum assured Rs.1, 00,000 and above. Double accident is allowed up to a maximum of Rs.10lakhs sum assured on payment of Re.1 per 1000 sum assured. Calculate yearly premium for Rs.15 lakhs sum assured with occupation extra of Rs.4 per thousand sums assured plus double accident benefit is allowed. a. b. c. d.

49,150 48,150 43,150 52,150

Solution: Investment Planning – 19Oct 07

69

Tabular premium Rebate allowed are yearly mode Rs.2 large sum assured Rs.3 Balance Balance X sum assured / 1000 Plus occupation extra 4 X 1500 Plus double accident benefit 1 per 1000 – since maximum double accident benefit allowed in up to Rs.10, 00,000 sum assured Total Yearly installment premium is Rs.49, 150.

Rs.33.10 (Rs.5.00) Rs.28.10 Rs.28.10 X 15, 00,000/1000 = Rs.42, 150 Rs.6000

Rs.1, 000 Rs.49, 150

Based on the above information and assuming a 6% per annum interest rate, the annual surrender cost index for each Rs.1,000 sum assured at the end of 20 years A. For Office 1 is greater than Office 2 B. For Office 2 is greater than Office 1 C. For Office 1 is equal to Office 2 D. Is not determinable 397. A group of 60000 persons each aged 45 years wishes to apply for term insurance for a one year period for a sum of Rs. 2,80,000. If mortality tables show that out of 80,00,000 people 40,000 die within a year, find the premium to be paid by each of the 60000 applicants. a. 1500 b. 1250 c. 1400 d. 2750

398. Calculate HLV to recommend adequate insurance cover; Mr. Satish, Age= 30 yrs Retirement age 65 years. He is senior Manager. His monthly salary is 10,00,000. he pays professional tax of rs.5,000 and income tax subject to allowable deductions i.e. tax paid Rs. 1,95,000 Reasonable self maintenance expenditure estimated Rs.1,00,000 p.a.; life insurance premium for self Rs. 20,000 with total sum assured Rs. 20,00,000. For wife and child he pays insurance premium of Rs. 13000 and Rs. 7000 respectively rate of interest assumed for capitalization of future income is at 10%. Adequate additional insurance recommended is a) Rs.65 lakh b) Rs.68 lakh c) Rs.88 lakh d) Rs.86 lakh 399. The following are statements concerning contracts of insurance. Identify the statement/s that is/are correct. I. Whenever the wording in an endorsement or rider is in conflict with the terms of the policy to which it is attached, the endorsement or rider takes precedence. II. One reason for exclusions in insurance policies is that the risks are covered by other insurance. A. I only B. II only C. I and II D. Neither I nor II 400. Mohan invested Rs. 420000 for 7 years @ 7% where it was compounded annually for the first 5years and quarterly for the last 2 years. What did he receive on maturity? a) 676774 b) 776774 Investment Planning – 19Oct 07

70

c) 931095 d) 609870

Quiz Inv Class Test 1 Type 1 one mark 401. The correlation co-efficient of two perfectly correlated assets is a. b. c. d.

_____

+1 -1 +0.5 -0.5

402. A zero coupon bond will have zero_______ risk a) b) c) d)

Interest Rate risk Inflation risk Reinvestment risk Default risk

403. Shyam has a one year old daughter. He wants to save for her higher education which shall begin at age 21. Which investment vehicle has the best potential for him to attain his goals?

a) b) c) d)

Derivative Instruments GOI Bonds Bank Fixed Deposits Equity Mutual Funds

404. ______________ is a form of Secured Loan a) a. b) c)

All of the above Loans against shares Credit Card Debt Holiday Loans

405. Beta is a term commonly used in the __________ a. b. c. d.

Art market Real Estate market Fixed Income Market Stockmarket

406. Bank Fixed Deposits eligible for 80 C deduction must have a minimum tenure of _______ years a) b) c) d)

3 4 5 6

407. ____________ bonds are eligible investments u/s 54 EC a) b) c) d)

NHAI NABARD REC Both A and C

408.__________ is an asset which is exempt from Long Capital Gains subject to certain conditions Investment Planning – 19Oct 07

71

a) b) c) d)

Equity Share Real Estate Art Commodities

409 _____________ are generally the preserve of the Wealthy class as compared to the Middle class a) b) c) d)

Fixed Deposits Mutual Funds Equity Shares Collectibles

410. The Mumbai Stock Exchange is a ____________ a) b) c) d)

Corporate entity Partnership Firm Association of Persons None of the above

411 ______________ funds have the lowest expense ratio a) b) c) d)

Equity Gilt Index Liquid

412 Your residence is treated as a ____________ asset a) b) c) d)

Personal Use Investment Near-Cash None of the above

413. The yield curve is usually _________________ a) b) c) d)

Upward sloping Downward sloping Flat Humped

414. Value investing involves purchasing stocks with __________ a) b) c) d)

Low P/E Ratios Low Book Values Low Dividend Yields Low Margin of safety

415. Narrow money does not comprise of _____________ a) Currency with the public b) Demand deposits with the banking system c) Other deposits with the RBI d) Time Deposits with banks 416. The rate at which commercial banks and other lending facilities can borrow short-term funds from the central bank is called the a) b) c) d)

Discount rate Repurchase Rate Reverse Repurchase Rate Prime Lending Rate

417. The effective interest rate earned per rupee _______ as the periods of compounding increase. Investment Planning – 19Oct 07

72

a) b) c) d) e)

Increases Decreases Remains same Decreases for some time and then increases Data insufficient

418. Sharpe ratio determines return per unit of _______________ a) b) c) d)

Diversifiable Risk Non-diversifiable risk Both of the above None of the above

419 Treynor ratio uses _______ as the divisor. a) b) c) d)

Beta Standard Deviation Variance Either b. or c.

420. Jensen‟s ratio helps in determining __________ a) b) c) d)

Alpha Beta Gamma Vega

421. SEBI is the Regulator for _____________ companies a) b) c) d)

Listed Unlisted Both A and B Foreign

422 An important characteristic of gilt securities are that they ___________. a) b) c) d)

are issued by the Central Government. all have terms to maturity that are 270 days are less. all tend to have large amounts of purchasing power risk. are issued by corporates

423. One standard deviation means that an observation will fall between the upper and lower bound ______ % of the time. a) b) c) d)

50.25% 68.30% 75.5% 95%

423. The statistical measure of covariance is known as _____________ a) b) c) d)

Standard Deviation Correlation coefficient. Variance Level of confidence

424. ________ is a measure of systematic risk a) Correlation coefficient b) Beta c) Alpha Investment Planning – 19Oct 07

73

d) Sharpe Ratio 425. An investment in equity shares of Russian companies is most probably, not subject to ________ a) b) c) d)

Currency Risk Political Risk Default Risk Liquidity Risk

426. Profits earned through trading exchange traded options are treated as ________ gains. a) b) c) d)

Short Term Capital Speculative Long Term Capital None of the above

427. The features of collectibles include ________ a) b) c) d)

The fact that they can be easily valued. The fact that they are traded on a Collectible Exchange The fact that they have the potential for great capital appreciation. The fact that common people can invest in them.

428. __________ is a widely used metric for measuring volatility of an investment. a) b) c) d)

Mean Median Standard Deviation Variance

429. The IRR method assumes that all cash flows are reinvested at __________ a) b) c) d)

The cost of capital The market rate of return The prevailing PLR The rate decreed by the RBI

430. A rating of AAA (ind) by Fitch indicates that the particular security is free from _________ risk a) b) c) d)

Interest rate risk Default risk Inflation risk Reinvestment risk

431. The YTM of a Bond is also known as its ______________ a. b) c) d)

Internal Rate of Return Holding period Return Current Yield None of the above

432. Equity Linked Savings Schemes eligible for Sec. 80 C benefits have a lock-in period of ____ years. a) 2 b) 3 c) 4 d) 5 433. The most appropriate investment for a highly risk averse investor aged 57 is __________ Investment Planning – 19Oct 07

74

a) b) c) d)

An index fund based on the Nifty. A diversified equity fund A Maharashtra State Government Bond A SBI Fixed Deposit

434 The thumb rule states that we should choose an investment with a _______ IRR. a) b) c) d)

Lower Higher Equal Data insufficient

435. Yield to Call is usually calculated for a ________ a) b) c) d)

Debt Instrument with an option Option free instrument Derivative instrument Data insufficient

436. The relationship between price and interest rate is _____________ a) b) c) d)

Linear Convex Random Concave

437. The following is not correct about a Gilt Mutual Fund. a) b) c) d)

They primarily invest in securities issued by the Central Government. They primarily invest in longer dated instruments. They are not subject to Capital Gains Tax if held for a period of over one year. They are more volatile as compared to liquid funds.

Type 2 – Two Marks 438. You are in the process of designing a portfolio of securities for your clients. While emphasizing the importance of diversification in the portfolio, you will explain that this should aim to ______________. a) b) c) d)

Completely eliminate both systematic and unsystematic risks. Completely eliminate systematic risks. Reduce unsystematic risks to the best possible extent Increase return at all costs.

439. When we take into account only negative deviations from the mean, we term it as ___________ a) b) c) d)

Standard Deviation Variance Semi-variance Mode

440. How many years (approx.) will it take for a sum of Rs.10000 to quadruple if the rate of return is 9% p.a. a) b) c) d)

18 16 14 12

441. If the post tax rate of return on an investment is 8% and the inflation rate is 5% the real rate of return is Investment Planning – 19Oct 07

75

a) b) c) d)

3.5% 3.0% 2.86% -3.0%

442. Geeta invests Rs. 2000 at the beginning of each month for 48 months. Her rate of return is 8% p.a. The investment‟s value at the end of the said period will amount to ___________ a) b) c) d)

1,10,000 1,12,700 1,15,000 1,11,500

443. Suppose you invest in 4 securities. Company A has an expected return of 20%. Company B 10%. Company C 12% and Company D 9%. You have invested Rs. 40,000. What more information is needed to find out the return on the portfolio? a) b) c) d)

Beta of these shares Proportion of investment Market Value of the investments None of the above

444. One standard deviation means that an observation will fall between the upper and lower bound ______ % of the time. a) b) c) d)

50.25% 68.30% 75.5% 95%

445.If Standard deviation is 8, the variance is equal to ________ a) b) c) d)

56 64 72 80

446. _________ distribution, indicates that an a particular investment has a probability of one potentially large loss as against several potentially small gains. a) b) c) d)

Lognormal Positively skewed Negatively skewed Normal

447. The P/E Ratio of the Sensex is 15. The earnings yield is ______ a) b) c) d)

5.50 6.25 6.67 7.05

448. A bank deposit of Rs. 25000 will earn an interest of Rs._____ at the end of one year, if it earns 10% p.a. compounded every month. a) 2599 b) 2617 c) 2745 d) 2799 Investment Planning – 19Oct 07

76

449. A bondholder buys a bond for Rs.105 and earns Rs.10 p.a. as interest. His current yield is ______ % a) b) c) d)

9.85 9.69 9.52 9.39

Solution = (10/105*100 = 9.52%) 450. A bondholder buys a bond maturing in one year for Rs. 90 and earns Rs. 5 per annum as interest. His Holding period yield is _______ % a) b) c) d)

15.85 16.67 17.25 18.19

Solution = (100-90) + 5 / 90*100 = 16.67% 451. John has an investment with an annual income of Rs.100 and current value of Rs.6,000. If the value of the market is expected to rise to Rs.7200 by end of 3 years, the approximate yield on the investment is _______ % a) 7.25 b) 7.58 c) 7.85 d) 8.02 Solution

future price current price number of years future price current price 2

annual income Formula : Approximate yield =

452. ____Losses suffered in stock market related transactions can be carried forward for upto ___ years. a) b) c) d)

Short Term Capital, 8 Speculative, 8 Long Term Capital, 8 Both a. and b.

453 Ram is an income seeking who is low on the risk curve. His Financial Planner suggests that he invest in a Company Fixed Deposit rated FB by CRISIL, which is offering a yield of 300 basis points over the FD rated FAAA. His advice is ________ a) b) c) d)

Appropriate because Ram is able to earn high interest. Appropriate because Ram is low on the risk curve. Inappropriate because Ram is low on the risk curve Inappropriate because Ram can earn even higher income by investing in a FD rated FC.

454. Senior Citizens Bonds offer _____ % interest and are eligible for _______ tax deduction a) b) c) d)

8.5, 80C 9.0, Nil 9.5, 80C 8.75, Nil

455. A company offers a rights issue of one for three for Rs.5 each. The present share price is Rs.13. If the share price does not change during the time of trading, what is the price after the rights are taken up? a) 11.00 b) 12.50 Investment Planning – 19Oct 07

77

c) 10.00 d) 10.50 Solution: 13*3 = 39 + 1*5 = 44. 44/4 = Rs. 11.00 456. A bondholder buys a bond maturing in two years for Rs. 120 and earns Rs. 15 per annum as interest. His YTM is _______ % a) b) c) d)

4.00 4.55 4.75 4.95

Solution:

Bond' s par value - Current bond price Number of years to maturity Bond' s par value Current bond price 2

Annual interest Yield to maturity (YTM)

Therefore YTM = {15 + (-20/2)} / {(100+120)/2} = 4.55 % 457. The net asset value (NAV) of a closed end fund has risen from Rs.52 to Rs.62. The fund was last quoted at Rs40, prior to the announcement of the increase in the NAV. If the fund were to trade at the same discount to NAV, estimate the new price of the fund? a) b) c) d)

48 52 54 57

Solution: The price is calculated as follows: (62 – 52)/52 = 19.2% 40 x 1.192 = 48

Investment Planning – 19Oct 07

78

Related Documents

Inv Plng
November 2019 18
Inv
July 2020 25
Inv
June 2020 18
Inv
October 2019 46
Inv
November 2019 37
Inv Minmax
November 2019 23